Vous êtes sur la page 1sur 147

Contents

MEANING ANALYSIS ....................................................................................................................... 12


ERROR ANALYSIS ............................................................................................................................. 13
POE ...................................................................................................................................................... 13
CHOICE A ....................................................................................................................................... 13
CHOICE B ....................................................................................................................................... 14
CHOICE C ....................................................................................................................................... 14
CHOICE D ....................................................................................................................................... 14
CHOICE E ....................................................................................................................................... 15
SAI NGU ................................................................................................................................................ 40

1. Students have been throwing away healthy meals made available in school cafeteria by The Los Commented [a1]: C
Angeles Unified School District and bringing their own junk food to school, and these forced the district
to bring back some of the foods it worked so hard to replace.

A. school, and these forced y c phi l li and m 2 mnh ko parrallel ko

B. schools, which forced

C. schools, forcing

Q#1 - WHY THE CORRECT ANSWER IS CORRECT

Students have been throwing away healthy meals made available in school cafeteria by The Los
Angeles Unified School District and bringing their own junk food to school and these forced the district
to bring back some of the foods it worked so hard to replace.

We will apply the e-gmat three-step process to solve this problem.

STEP 1: MEANING ANALYSIS

This sentence says that Los Angeles Unified School District added healthy meals to the school cafeteria.
But students have been throwing away these healthy meals and have been bringing their own junk
food. As a result of these actions, the district was forced to bring back some of the foods it worked so
hard to replace.

STEP 2: ERROR ANALYSIS

Lets now split the sentence into clauses to spot the errors:
Students have been throwing away healthy meals made available in school cafeteria by The Los
Angeles Unified School District and bringing their own junk food to school
and these forced the district to bring back some of the foods
o it worked so hard to replace.

In the second clause, pronoun these have been used to refer the actions of throwing healthy meals
and bringing junk food. This usage is not correct because these is a pronoun. It can only refer to a
noun or another pronoun. It cannot be used to refer to an action or a clause.

STEP 3: POE

A. school and these forced: Incorrect for the reason discussed during error analysis.

B. schools, which forced: Incorrect. Notice that which is a noun modifier that refers to the preceding
noun or noun phrase. In this choice, which is referring to schools. This is illogical. Also notice that
which is a relative pronoun that abides by the same rules for pronoun. Hence, which cannot be
used to present the result of the preceding clause.

C. Schools, forcing: Correct. The verb-ing modifier forcing appears in this sentence preceded by a
comma. This means that forcing is modifying the preceding clause. Per the context of the sentence,
comma + forcing, in this choice, is correctly presenting the result of the preceding clause.

2. In three months, biologist Glauco Machado gathered enough information about large numbers Commented [a2]: A
of a relatively unstudied order of arachnids to persuade an ant specialist at the university to advise him
and to publish his first scientific paper.

A. arachnids to persuade an ant specialist at the university to advise him and to publish

B. arachnids, persuading an ant specialist at the university to advise him and publishing

C. arachnids persuading an ant specialist at the university to advise him and publishing [an order
of arpachnids ko th persuade c con ngi]

In three months, biologist Glauco Machado gathered enough information about large numbers of a
relatively unstudied order of arachnids to persuade an ant specialist at the university to advise him and
to publish his first scientific paper.
A. arachnids to persuade an ant specialist at the university to advise him and to publish
B. arachnids, persuading an ant specialist at the university to advise him and publishing
C. arachnids persuading an ant specialist at the university to advise him and publishing

Lets use the tools to determine whether verb-ing modifier makes sense.

STEP 1: (Intended) MEANING ANALYSIS

The sentence says that in three months Machado gathered enough information about a huge number of
comparatively unstudied order of arachnids. He gathered all information with two purposes in mind:

a. he wanted to persuade an ant specialist at the university to advise him


b. he wanted to publish his first scientific paper

STEP 2: ERROR ANALYSIS

In three months, biologist Glauco Machado gathered enough information about large numbers of a
relatively unstudied order of arachnids to persuade an ant specialist at the university to advise him and
to publish his first scientific paper.

This sentence has just one subject-verb pair, meaning it has only one clause. The SV pair is accounted
for. The purposes of gathering all the information have been correctly written in to verb form. These
two purposes are also correctly joined with and. Hence, there is no error in this sentence. The
sentence is correct as is.

STEP 3: POE

Let us now do the POE to see what makes the other two choices incorrect.

A. arachnids to persuade an ant specialist at the university to advise him and to publish: Correct as we
discussed during error analysis.

B. arachnids, persuading an ant specialist at the university to advise him and


publishing: Incorrect. Here both the verb-ing modifiers are preceded by comma, implying that they
modify the entire preceding clause. Hence, now the sentence means that Machado gathered all the
information and this action resulted into two things:
a. he persuaded the ant specialist, and
b. he published his first scientific papers.
This is certainly not the intended meaning of the original sentence. The original sentence talks about
purpose. Per this choice, Machados gathering information actually led to the persuasion of the ant
specialist and the publication of the first scientific paper. Hence, this choice is grammatically correct
but certainly alters the intended meaning and is thus incorrect.

C. arachnids persuading an ant specialist at the university to advise him and publishing: Incorrect. In
this choice, the verb-ing modifiers appear without comma. Here, both persuading and publishing
modify the preceding noun arachnids. Now, per this choice, the sentence means that Machado
collected information on certain arachnids and these arachnids did the jobs of persuading the ant
specialist and publishing first scientific paper. This is absolutely illogical. This is a case where verb-ing
modifier without a comma does not make sense.

Mi ngi vo xem cu ny gip mnh nh:


In the past several years, astronomers have detected more than 80 massive planets, most of them
as large or larger than Jupiter, which circle other stars.
(A) most of them as large or larger than Jupiter, which circle
(B) most of them as large or larger than Jupiter and circling
(C) most of them at least as large as Jupiter, circling
(D) mostly at least as large as Jupiter, which circle
(E) mostly as large or larger than Jupiter, circling
OA: C
im mnh thc mc l "circling".
Vit li cu ny: astronomers have detected more than 80 massive planets,xxx, circling...
Do "xxx = most of them as large as jupiter" l nonessential modifier nn c th b i, v circling th ang c
hiu b ngha cho main clause, nhng nh th th astronomers ko l ch ng ca hnh ng Circling, m v
ngha cng ko phi l consequences hay lm r ngha cho mnh trc.
Ron gii thch l circling b ngha cho jupiter
--> cu hi cu mnh: trng hp no m Comma + Ving ng cui cu l b ngha cho mnh trc, trng
hp no l b ngha cho Noun pha trc ? so confused.
Thanks mi ngi
Trng Hiu Nguyn Circling lc ny ang modify cho 80 massive planets ch ko modify cho main clause nh.
Huyen Nguyen vy nu modify cho 80 planets th e vn cha r khi no "comma + ving" b ngha cho c clause v khi no b
ngha cho Noun trc n ah ?
Trng Hiu Nguyn Verbing sau comma lun modify cho clause. Cn trng hp ny th ko phi l sau comma.
Huyen Nguyen anh l comma + ving phi ng ngay sau 1 clause, cn y cn sau 1 on b sung cho 80 planet nn tt c
phn sau u b sung cho 80 planet ?
Trng Hiu Nguyn Uhm, 2 ci comma ny function ca n l introduce nonessential modifier, ko phi start ci verbing
modifier, nn ko tnh. Circling vn ang modify cho 80 massive planets.
Huyen Nguyen Qa l vi diu :))))
Trng Hiu Nguyn Cu ny trc cng c mt bn hc vin pht hin ra ri, search ko ra gii thch no n c, nh phi au
u ngh mt lc mi ra :))) o lm.
Huyen Nguyen vy m my hm nay e vn dn vt ngh mnh "ngu" lm
Diu Hng Vy nu ko c nonessential modifier th cu s l :astronomers have detected more than 80 massive planets circling
others stars sao, hay nh.
Trng Hiu Nguyn ng ri

GMAT Prep Question:

In the past several years, astronomers have detected more than 80 massive planets, most of them as
large or larger than Jupiter, which circle other stars.
A. most of them as large or larger than Jupiter, which circle
B. most of them as large or larger than Jupiter and circling
C. most of them at least as large as Jupiter, circling
D. mostly at least as large as Jupiter, which circle
E. mostly as large or larger than Jupiter, circling
Let us now apply the e-gmat three-step process to solve this one. We will begin with the first step that
is:

MEANING ANALYSIS: The sentence says that in the past few years, astronomers have detected more
than 80 massive planets. These massive planets are either as large as Jupiter or larger than Jupiter.
These massive planets also circle other stars.

ERROR ANALYSIS:

In the past several years, astronomers have detected more than 80 massive planets, most of them as
large or larger than Jupiter,
which circle other stars.

This sentence uses a mix of two idioms as X as and X larger than in the form of as large or
larger than. This is incorrect. We must use the idiom correctly in its entirety to present the correct
comparison.
Notice the modifier which circle. This relative pronoun modifier modifies the noun 80 massive
planets. Some of you may wonder that which circle modifies the closest noun Jupiter. This is
not correct. And this is where logical meaning and understanding the sentence structure comes to our
aid.

Notice that most of them as large or larger than Jupiter is a modifier that provides additional
information about the 80 massive planets. This modifier is encapsulated in a comma pair. So even if we
remove this modifier from the sentence, the meaning will not be altered. So for the sake of our
discussion, lets remove this modifier:
In the past several years, astronomers have detected more than 80 massive planets, most of them as
large or larger than Jupiter,which circle other stars.

Now as you can see, which circle modifies the noun phrase 80 massive planets. Remember that
the comma that precedes which does not separate which but is part of the comma pair that
separates the modifier "most of them" from the rest of the sentence.
We will now find the correct choice from the remaining 4 choices.

POE:

A. most of them as large or larger than Jupiter, which circle: Incorrect for reasons discussed above.

B. most of them as large or larger than Jupiter and circling: Incorrect.


i. This choice repeats the idiom and parallelism errors of choice A.
ii. A modifier is not separated from the entity it modifies by using and.

C. most of them at least as large as Jupiter, circling: Correct. This choice corrects the error of choice A
by using the correct idiom at least as large as Jupiter to convey the intended meaning.
Furthermore, notice the use of modifier - circling . This modifier modifies the noun entity that
precedes it more than 80 massive planets. As we discussed in the sentence structure discussion for
choice A, this modifier does not modify the immediately preceding noun "Jupiter". This is a verb-ing
modifier that is not separated from the preceding clause by a comma and hence modifies the preceding
noun entity.
At this time we should also pay attention to the logical meaning of the sentence. We have determined
already that circling modifies 80 massive planets from grammatical standpoint. From logical
standpoint also, circling must modify 80 massive planets because it is a universal fact that Jupiter
revolves around the Sun and no other star. Hence, there is no logical way that circling can modify
Jupiter. It should modify 80 massive planets.

D. mostly at least as large as Jupiter, which circle: Incorrect. Use of mostly is incorrect here. Now it
is an adverb that refers to the adjective following it at least as large as. This is non-sensical.

E. mostly as large or larger than Jupiter, circling: Incorrect.


i. Repeats the same mostly error of choice D.
ii. Repeats the idiom error of choice A.

Applying RULE SET #1 on OG 12 #47

Five fledgling sea eagles left their nests in western Scotland this summer, bringing to 34 the number of
wild birds successfully raised since transplants from Norway began in 1975.

(A) bringing
(B) and brings
(C) and it brings
(D) and it brought
(E) and brought

We will solve this question, using e-gmat three-step process.

STEP 1: MEANING ANALYSIS


The sentence says that this summer, five fledgling sea eagles left their nests in western Scotland. This
action brought to 34 the total number of wild birds that have been successfully raised since transplants
from Norway started in 1975.

STEP 2: ERROR ANALYSIS

Let us split the sentence into clauses to spot the errors.


Five fledgling sea eagles left their nests in western Scotland this summer, bringing to 34 the number
of wild birds successfully raised
o since transplants from Norway began in 1975.

The only word underlined in this sentence is the verb-ing modifier bringing. This modifier appears
after the main clause preceded by a comma. This means it modifies the preceding clause. The use of
this modifier is correct here because per the context, it presents the result of the preceding clause -
one of the functions of the verb-ing when preceded by a comma. Hence, there is no error in this
sentence. This sentence is correct as is.

STEP 3: POE

Through this step we will find out why the other answer choices are incorrect.
(A) bringing: Correct as explained during error analysis.

(B) and brings: Incorrect. brings is not the correct verb tense to refer to an action that took place in
the past. Furthermore, use of comma + and to connect two entities in a list here two verbs of the
same subject - is not correct.

(C) and it brings: Incorrect.


i. Singular pronoun it neither can refer to plural eagles nor can refer to the preceding clause. A
pronoun can only refer to a noun or another pronoun.
ii. Repeats the same verb tense error as in Choice B.

(D) and it brought: Incorrect. Repeats the same pronoun error as in Choice D.

(E) and brought: Incorrect. Use of comma + and to connect two entities in a list here two verbs of
the same subject - is not correct.

Hi Pavan, Commented [a3]: Cu ny vn hi kh hiu

On account of a law passed in 1933, making it a crime punishable by imprisonment that a United States
citizen hold gold in the form of bullion or coins, immigrants found that on arrival in the United States
they had to surrender all of the gold they had brought with them.

A. On account of a law passed in 1993, making it a crime punishable by


imprisonment that a United States citizen hold
B. With a law passed in 1933 that makes it a crime punishable by imprisonment that a United States
citizen hold
C. A law passed in 1933 that made it a crime punishable by imprisonment for a United States citizen
holding
D. Because of a law passed in 1933 making it a crime punishable by imprisonment for a United States
citizen to hold
E. Due to a law being passed in 1933 that makes it a crime punishable by
imprisonment for a United States citizen to hold

The sentence does not convey the meaning so clearly because of the construction. So lets first
understand what the sentence means. A law was passed in 1933. According to this law, it was a crime
punishable by imprisonment for a US citizen to possess gold in the form of bullion or coins. Because of
this law, immigrants found that they had to surrender all the gold they had brought with them on the
arrival in the US.

Now lets do the error analysis:


1. a crime that is not an idiomatic expression. We need to say a crime for a US citizen.
2. Accordingly, a US citizen hold gold will change into a US citizen to hold gold.

The verb-ing modifier here appears after law passed in 1933. Now passed in 1933 is a verb-ed
modifier that is modifying law. The verb-ing modifier is also modifying law, a slightly far away
noun. As such, we dont have modification issue here.

POE:

A. On account of a law passed in 1993, making it a crime punishable by imprisonment that a United
States citizen hold: Incorrect for the reasons stated above.

B. With a law passed in 1933 that makes it a crime punishable by imprisonment that a United States
citizen hold: Incorrect.
1. The sentence now seems to suggest that the immigrants arrived in the US with the law.
2. All the errors of Choice A are repeated.

C. A law passed in 1933 that made it a crime punishable by imprisonment for a United States citizen
holding: Incorrect.
1. There is no verb for the subject A law. We have a fragment here.
2. holding should be to hold.

D. Because of a law passed in 1933 making it a crime punishable by imprisonment for a United States
citizen to hold: Correct. This choice makes the sentence structure correct so that the meaning is
clear.

E. Due to a law being passed in 1933 that makes it a crime punishable by imprisonment for a United
States citizen to hold: Incorrect.
1. Use of Due to is incorrect.
2. Use of being passed is not correct.

Take Away:
1. The sentence structure should be such that it conveys the intended meaning clearly.
2. Always use correct idiomatic expressions for correct and clear meaning of the sentence.
3. Modifiers must make sense with the entities they modify.

For the farmer who takes care to keep them cool, providing them with high-energy feed, and milking
them regularly, Holstein cows are producing an average of 2,275 gallons of milk each per year.

(A) providing them with high-energy feed, and milking them regularly, Holstein cows are producing
(B) providing them with high-energy feed, and milked regularly, the Holstein cow produces
(C) provided with high-energy feed, and milking them regularly, Holstein cows are producing
(D) provided with high-energy feed, and milked regularly, the Holstein cow produces
(E) provided with high-energy feed, and milked regularly, Holstein cows will produce
Hi Pavan,
For the farmer who takes care to keep them cool, providing them with high energy feed and milking
them regularly, Holstein cows are producing an average of 2275 gallons of milk each year.

Since the meaning of this sentence is not too difficult to understand, let us get to error analysis. I
would just like to add here that this question pertains more to parallelism and verb-tense rather than
verb-ing modifiers.

1. Verb-ing modifiers do not make sense in this sentence because the farmers do not provide the cows
with high energy feed and milk them to keep them cool. These verb-ing modifiers do not make sense to
present additional information. Also, they cannot provide the results of the preceding clause. Doing so
will make the sentence illogical. So use of verb-ing modifiers is incorrect here.

2. This part is very unusual of official questions, but we have this one off-beat parallel list here. The
farmers takes care:
a. To keep them cool
b. To keep them provided with high energy feed, and
c. To keep them milked regularly.

This list could be made simple by saying farmers take care to keep them cool, provide them with high
energy feed, and milk them regularly. But the author has decided to make it complicated so lets keep
it that way.
So certainly, the entities in the list are not parallel.

3. Usage of present continuous are producing is incorrect here.

Now lets do the POE:

A. providing them with high energy feed and milking them regularly, Holstein cows are
producing: Incorrect for the reasons stated above.

B. providing them with high energy feed , and milked regularly, the Holstein cow
produces: Incorrect.
1. Verb-ing providing makes the list non-parallel.
2. Singular cow cannot refer to plural pronoun them.
3. The sentence is talking about general Holstein cows and not a particular Holstein cow. So the
Holstein cow is incorrect and subsequently the verb used for this subject too.

C. provided with high energy feed, and milking them regularly, Holstein cows are
producing: Incorrect.
1. Verb-ing milking makes the list non-parallel.
2. Present continuous are producing is incorrect.

D. provided with high energy feed, and milked regularly, the Holstein cow
produces: Incorrect. Same pronoun-antecedent number agreement error as in choice B.

E. provided with high energy feed ,and milked regularly, Holstein cows will
produce: Correct. Notice that this sentence is in if then construction, and hence use of will
produce is correct.
Scientists have recently discovered what could be the largest and oldest living organism on Earth, a
giant fungus
that is an interwoven filigree of mushrooms and root-like tentacles spawned by a single fertilized spore
some
10,000 years ago and extending for more than 30 acres in the soil of a Michigan forest.

A. extending
B. extends
C. extended
D. it extended
E. is extending

Lets first understand the sentence structure of this question:

1. Scientists have recently discovered


2. what could be the largest and oldest living organism on Earth, a giant fungus
3. that is an interwoven filigree of mushrooms and rootlike tentacles
a. spawned by a single fertilized spore some 10,000 years ago
b. and extending for more than 30 acres in the soil of a Michigan forest.

So as you can see, there are three modifiers for fungus


1. that is an interwoven filigree - Tells what is this fungus
2. spawned by... - Provides a characteristic
3. extending for...- Provides a characteristic

Spawned and extending are parallel because of the context of the sentence. The author wants to
establish two properties of fungus
1) it was spawned by single fertilized spore - This happened in the past.
2) it extends for more than 30 acres - this is true in the current time frame as well.

So this is logical parallelism. Now that we know that these have to be parallel, we can try to make
them grammatically parallel by making them consistent with each other. But in this case we cannot
make both verb-ed or both verb-ing modifiers since then the meaning will be distorted. Lets check that
-
1. fungus spawned by single...and extended for more than...
The second modifier now changes the intended meaning since it no longer implies that the fungus still
extends...

2. fungus spawning by single fertilized spore and extending for...


Now first modifier changes the meaning since it now implies that fungus is still in the process of
spawning - which is non-sensical.

Thus, the context of this sentence requires that verb-ed modifier be parallel to verb-ing modifier.

In this case, it does not make sense to say that tentacles are spawned by a single fertilized spore. It
only makes sense to say that the fungus was spawned by a single fertilized spore.
Beatrix Potter, in her book illustrations, carefully coordinating them with her narratives, capitalized on
her keen observation and love of the natural world.

(A) Beatrix Potter, in her book illustrations, carefully coordinating them with her narratives,
(B) In her book illustrations, carefully coordinating them with her narratives, Beatrix Potter
(C) In her book illustrations, which she carefully coordinated with her narratives, Beatrix Potter
(D) Carefully coordinated with her narratives, Beatrix Potter, in her book illustrations
(E) Beatrix Potter, in her book illustrations, carefully coordinated them with her narratives and

Hi Shraddha & E-GMAT Team,

Mine query is related to OG-12, #38, Please find the question below.

In 1850 Lucretia Mott published her Discourse on Women, arguing in a treatise for women to have equal
political and legal rights and for changes in the married womens property laws.

A. arguing in a treatise for women to have equal political and legal rights
B. arguing in a treatise for equal political and legal rights for women
C. a treatise that advocates womens equal political and legal rights
D. a treatise advocating womens equal political and legal rights
E. a treatise that argued for equal political and legal rights for women

Between A & E, there is only a change in a modifier, could you please elaborate why B is wrong,
i think publishing a book and then the modifier "arguing" can be treated as one of the outcomes or the
motive behind publication. Please correct me if i'm wrong?

Thanks

Hi Nitinaka19,

Analysis of Choice A: The sentence tells us that Lucretia Mott published "Discourse on Women". Then
we have a Comma + Verb-ing "arguing..." that modifies the preceding action. Now Comma + Verb-ing
modifiers perform two functions:

1. presents the "HOW" aspect of the preceding action - In this scenario it does not make sense to say
that Mott published "Discourse on Women" by arguing... for women...".
2. presents the "RESULT" of the preceding action - Even this function doe snot make sense because
"arguing for women rights" was not the result of publishing "Discourse on Women".

Also, this choice fails to establish that actually "Discourse on Women" is the treatise that Mott
published. The phrase "in a treatise" suggests that Mott argued for women rights in some treatise other
than "Discourse on Women".

Analysis of Choice E: This choice employs Noun + Noun Modifier that can refer to any entity in the
preceding clause. Here, "a treatise" = Noun and "that argued for..." = Noun Modifier. Together this Noun
+ Noun Modifier modifies "Discourse on Women". It gives more information about this entity. So
"Discourse on Women" is a treatise that did something. This Choice CLEARLY communicates the
meaning of the sentence. Hence, Choice E is the correct answer.

As a result of record low temperatures, the water pipes on the third floor froze, which caused the
heads of the sprinkler system to burst, which released torrents of water into offices on the
second floor.

A) which caused the heads of the sprinkler system to burst, which released torrents of water
B) which caused the heads of the sprinkler system to burst and which released torrents of water
C) which caused the heads of the sprinkler system to burst, torrents of water were then released
D) causing heads of sprinkler system to burst, then releasing torrents of water
E) causing heads of the sprinkler system to burst and release torrents of water

As a result of record low temperatures, the water pipes on the third floor froze, which
caused the heads of the sprinkler system to burst, which released torrents of water into
offices on the second floor.

MEANING ANALYSIS
As a result of record low temperatures, the water pipes on the third floor froze,
The sentence starts by presenting the result of an event. It says that in some place,
because of the unusually low temperatures, the water in the pipes on a certain floor of a
building froze.
o which caused the heads of the sprinkler system to burst,
This part presents the result of the event mentioned earlier. It tells us that because of
the resulting pressure from the freezing of the water in the pipes, the water-sprinklers
burst. Reading further:
which released torrents of water into offices on the second floor.
Finally, we get to know of another event in the series. We find out that the bursting of
the sprinklers forcefully released a lot of water in the offices situated on the floor
beneath the one on which the pipes were located.
To sum up, the sentence presents a chain of effects caused by a particular event. First,
the unusually low temperatures in a certain place led to the freezing of water in the
pipes on a certain floor in a building. This event caused the water sprinklers to burst,
forcefully dispensing a huge amount of water in the offices on the floor beneath.

ERROR ANALYSIS

Modifier Errors

Both the instances of the relative pronoun modifiers which are incorrect in the sentence.
The sentence intends to present the results of two particular actions. But it uses noun
modifier which for both the purposes. Because neither of the instances of which in the
sentence has a corresponding noun entity to modify, the sentence has modifier errors.

One of the ways to rectify this error is to use comma + verb-ing modifier to present the
result of the preceding action. For instance, the relevant portion of the sentence can be
written as:

the water pipes on the third floor froze, causing the heads of the sprinkler system
to burst,

POE

CHOICE A
Incorrect
This choice has the errors pointed out in the error analysis.

CHOICE B
Incorrect

Modifier Errors

In this choice, the relative pronoun modifier which has been used in two instances, and
in both these instances, which has been incorrectly used to modify the action of freezing.

CHOICE C
Incorrect

1. Modifier Error

This choice repeats the modifier error of choice A by repeating the first usage of which.

2. Structure Error

The independent clause (IC) torrents of water were then released is connected to
the preceding IC the water pipes froze just by a comma. This structure is
ungrammatical. Two ICs must be joined with comma + FANBOYS.

CHOICE D
Incorrect

Modifier Error

This choice employs the comma + verb-ing modifiers causing and then releasing.
Use of the first comma + verb-ing modifier causing is correct because it correctly
modifies the action of the preceding clause and also makes sense with the subject of the
modified clause the water pipes on the third floor. Per this modification, the water pipes
on a certain floor froze, and this action caused the heads of the sprinkler to burst.

However, use of the second comma + verb-ing modifier then releasing is not correct as
the modification illogically conveys that because the water pipes froze, torrents of
water were released. From the meaning analysis we know that water was released
because the sprinklers burst.

CHOICE E
Correct

This indeed is the correct answer choice. While the comma + verb-ing modifier
causing correctly presents the result of water pipes freezing, the two parallel actions
to burst and (to) release clearly convey that the sprinklers burst and dispensed
water because the water pipes froze.

Hi egmat,

I purchased egmat course some timeback; however, it got expired now. Hence I can't ask my doubt on
the official site.
I have just one doubt in one concept; please clarify it. I am interpreting something in a wrong way.
Please help !

Joe became the CFO of the company, increasing his pay significantly.

The above sentence is incorrect because "increasing" does not make sense with the subject.

For the below sentence from OG,(https://e-gmat.com/blogs/ogvr-1637/) I read the explanation(for


this ques) given on the official page of egmat, but it does not explicitly discuss the problem I am
facing

As a result of record low temperatures, the water pipes on the third floor froze, which caused the
heads of the sprinkler system to burst, which released torrents of water into offices on the second
floor.
the correct answer is -- causing heads of the sprinkler system to burst and release torrents of water

causing +--- should make the sense with water pipes(subject of the previous clause), but it does
not make sense to me.

How can water pipe cause the heads of the sprinkler system to burst?

An excerpt from Egmat page on choice D of this ques, but still I don't understand how can water pipe
cause the heads of sprinkle system to burst

This choice employs the comma + verb-ing modifiers causing and then releasing. Use of the first
comma + verb-ing modifier causing is correct because it correctly modifies the action of the
preceding clause and also makes sense with the subject of the modified clause the water pipes on the
third floor. Per this modification, the water pipes on a certain floor froze, and this action caused the
heads of the sprinkler to burst.

Hi There,

Thank you for your query. And I apologize for the delay in responding to you.

The problem that you are facing is quite common. Many a times, it does appear to us that the comma +
verb-ing modifier does not make sense with the doer of the modified action. The confusion takes place
because we try to associate the doer of the modified action with the action denoted by the modifier
independently. In a way, we turn the comma + verb-ing modifier in a verb for the doer of the action
and apply the SV makes sense rule on the pair. This approach is incorrect.

We must always bear it in our mind that the action denoted by the comma + verb-ing modifier factors
in the action that it modifies. While presenting additional information about the action it modifies, the
modifier associates with the doer of the modified action.

The question that we should ask while evaluating the correct usage of the comma + verb-ing modifier is
who or whose action is responsible for the action denoted by the verb-ing modifier. If the answer is
yes, then the sentence uses the comma + verb-ing modifier correctly.

Lets put this test into action. Lets first apply this test on the Joe sentence. Following is the
sentence:
Joe became the CFO of the company, increasing his pay significantly.
The question we ask:
o Did the action done by Joe increase his pay? No.
Hence, in the above-mentioned sentence, usage of comma + verb-ing modifier is incorrect.

Now lets apply this test on the official question you have doubt about.
As a result of record low temperatures, the water pipes on the third floor froze, which caused the
heads of the sprinkler system to burst, which released torrents of water into offices on the second
floor.
The question we ask:
o Did the action done by the water pipes caused the heads of the sprinklers to burst? Yes.

Hence, in the above-mentioned sentence, usage of comma + verb-ing modifier is correct.

Hi!! In view of the above article, could you please advice how the following is correct:

The original building and loan associations were organized as limited life funds, whose members made
monthly payments on their share subscriptions and then took turns drawing on the funds for home
mortgages.

According to your article, "drawing on the funds for home mortgages" should modify "turns", but that
does not seem to be the case.

Kindly advice.

Hi Mohish,

First of all thanks for posting this question. It is a great question that exemplifies yet again that we
should understand the context first and then apply our knowledge of grammar rules accordingly. Lets
see this in action on this question:

The original building and loan associations were organized as limited life funds, whose members made
monthly payments on their share subscriptions and then took turns drawing on the funds for home
mortgages.

This is officially correct choice of Og12#114. The context of the sentence tells us that the members
made monthly payments on something and they also took turns in doing something. So the context
actually associates drawing to turns. i.e. the members took turns in drawing on the funds for home
mortgages.

However, drawing is not a verb-ing modifier here. Why? Because turns are not drawing anything.
We cannot say: turns that drew on the funds This is illogical. From the context we know that the
members took turns in doing something.

So here, verb-ing form of word draw - drawing is actually functioning as a noun. This noun with an
ing form is called gerund.

For what action did members take turns? Members took turns in drawing funds. The expression here
means that one by one the members drew on funds. Take this sentence for example:

Before the competition, the contestants take turns swimming in the pool.

This sentence clearly says that before the swimming competition, the contestants swim in the pool one
by one. They take turns swimming in the pool. Use of swimming is correct here. And here,
swimming is not a modifier. It is a noun. Same is the case with the official sentence where drawing
is not a verb-ing modifier (adjective) that modifies the preceding noun. It is a noun entity.

Take Away
1. Verb-ing word can play multiple roles
a. Act as a verb continuous tense when preceded by is/are/am/was/were (not discussed here)
b. Act as a modifier discussed in part 1 and 2 of articles.
usage-of-verb-ing-modifiers-135220.html
udsage-of-verb-ing-modifiers-continued-135567.html
c. Act as a noun also called as gerund discussed in this post.

2. Always understand the meaning of the sentence to determine the function of each entity in the
sentence.
Hi Shraddha/Payal,

Could you please help me in understanding the below question

The intricate structure of the compound insect eye, having hundreds of miniature eyes called ommatidia,
help explain why scientists have assumed that it evolved independently of the vertebrate eye.

A having hundreds of miniature eyes called ommatidia, help explain why scientists have assumed that
it
B having hundreds of miniature eyes that are called ommatidia, helps explain why scientists have
assumed that they
C with its hundreds of miniature eyes that are called ommatidia, helps explain scientists' assuming that
they
D with its hundreds of miniature eyes called ommatidia, help explain scientists' assuming that it
E with its hundreds of miniature eyes called ommatidia, helps explain why scientists have assumed
that it

I know, that the comma + ing should modify the previous clause. Here, we don't have a clause before
comma+ing modifier, hence the usage is incorrect, however OG says that the wording suggests that the
intricate structure has miniature eyes. Could you please help me out. Also, does comma+ with modifier
modifies closest noun or it can modify clause as well.

Thanks
Himanshu

Hi Himanshu,

The intricate structure of the compound insect eye, having hundreds of miniature eyes called
ommatidia, help explain why scientists have assumed that it evolved independently of the vertebrate
eye.

It is true that comma + verb-ing modifies the preceding clause. But that happens when comma + verb-
ing modifier is placed after a clause. There may be instances where you will see comma + verb-ing
modifier is placed after the subject. In that case, the comma + verb-ing modifier modifies the subject.

Grammatically, in this OG 13#7 problem, having is modifying the subject The intricate structure of
the compound eye after which it has been placed. However, this leads to illogical meaning. Now the
sentence means that the intricate structure has hundreds of miniature eyes. Hence, we certainly have
modifier error here.
Hi debayan,

I kind of got it that you think that it refers to The intricate structure of the compound insect eye.
So per your answer, the sentence says that the intricate structure of the compound insect eye, having
hundreds of miniature eyes called ommatidia, helps explain why scientists have assumed that the
structure evolved independently of the vertebrate eye.

Now, tell me does that make sense? Can a structure evolve? That to independently of the vertebrate
eye?

So, pronoun it does not refer to the structure. This it refers to the compound insect eye. And
the scientists now have the reason to explain why they assume that the compound insect eye evolved
independently of the vertebrate eye. This is the logical meaning of the sentence.

Now, study the following OG 13#25 problem,

Neuroscientists, having amassed a wealth of knowledge over the past twenty years about the brain
and its development from birth to adulthood, are now drawing solid conclusions about how the human
brain grows and how babies acquire language.

In this sentence, usage of having is absolutely correct. It is placed after the subject neuroscientists
and hence correctly modifies that entity. The meaning here is that neuroscientists have amassed a
wealth of knowledge The modification conveys the logical meaning.

Now lets talk about the comma + with modifiers. So here we are talking about prepositional
phrases. with modifiers are very versatile modifiers. They can modify either the preceding clause or
the preceding nouns. What they modify actually depends on the context of the sentence and the
wording of the modifier itself.

Bihar is India's poorest state, with an annual per capita income of $111.

In this sentence, with modifier actually modifies the preceding noun. With modifier in this sentence
has the following sense. Notice how which has can be understood to replace with.
Indias poorest state, which has an annual per capita income of $111

However, this modifier can be understood to modify the subject of the clause as well because of the
nature of the verb - is. This is a linking verb, which establishes the following relationship:
Bihar = Indias poorest state.

Now lets consider a scenario in which with modifier modifies the preceding clause. This is a
GMATPrep Question. You can find the detailed solution of this question at this link.
Visitors to the park have often looked up into the leafy canopy and seen monkeys sleeping on the
branches, with arms and legs hanging like socks on a clothesline.

The comma +with modifier above modifies the preceding clause. In essence this sentence can be
written as two separate sentences:

1. Visitors to the park have often looked up into the leafy canopy and seen monkeys sleeping on the
branches.
2. Their arms and legs are hung like socks on a clothesline.

So sentence 2 has been converted into with modifier. This modifier extends the thought of the
preceding clause by providing a detail supporting it.

Now if the above sounds very complicated, then do not worry about it. As long as you know that these
modifiers are versatile and hence can modify preceding clause and preceding nouns, you would be fine.
Let the meaning of the sentence guide you. You should understand the meaning of the sentence and
ensure that one of these roles fit well.
Hi @pavanpuneet,

There are numerous products in order that they might use to seal a bottle of wine including a cork, a
screwcap, and a newly released item called the Vino-Lok that combines the elegance of a cork with the
practicality of a screwcap.

A. in order that they might use to seal a bottle of wine including a cork, a screwcap, and a

B. that are used to seal a bottle of wine including a cork, a screwcap, and a

C. so as they might use to seal a bottle of wine including a cork, a screwcap, and a

D. so that there could be used to seal a bottle of wine including a cork, a screwcap, and a

E. such that they could use to seal a bottle of wine including a cork, a screwcap, and a

Now lets look at the sentence with the correct answer choice B:

There are numerous products that are used to seal a bottle of wine including a cork, a screwcap, and a

newly released item called the Vino-Lok that combines the elegance of a cork with the practicality of
a screwcap.

The first thing to notice is that including does not follow the rules of the verb-ing modifiers.
Including always refers to a noun entity and needless to say a logical noun entity. Generally,
including is placed next to the noun entity it refers to. But that is not the case always.

This sentence is an example of that scenario. In this sentence, including is correctly modifying
numerous products. It is jumping over the that clause modifier that modifies the same entity. Now
that clauses are always placed next to the entity that they modify. So here, it is not possible to
change the placement of that clause. Since including is pretty flexible, it has been separated from
numerous products by adding a modifier in between. This usage is absolutely correct.
In the Question above, though I got the right answer, I would like to understand what is "including"
modifying here? In other words, is it used as a modifier? Yes/No.

Hi @pavanpuneet,

There are numerous products in order that they might use to seal a bottle of wine including a cork, a
screwcap, and a newly released item called the Vino-Lok that combines the elegance of a cork with the
practicality of a screwcap.

A. in order that they might use to seal a bottle of wine including a cork, a screwcap, and a

B. that are used to seal a bottle of wine including a cork, a screwcap, and a

C. so as they might use to seal a bottle of wine including a cork, a screwcap, and a

D. so that there could be used to seal a bottle of wine including a cork, a screwcap, and a

E. such that they could use to seal a bottle of wine including a cork, a screwcap, and a

Now lets look at the sentence with the correct answer choice B:

There are numerous products that are used to seal a bottle of wine including a cork, a screwcap, and a
newly released item called the Vino-Lok that combines the elegance of a cork with the practicality of a
screwcap.

The first thing to notice is that including does not follow the rules of the verb-ing modifiers.
Including always refers to a noun entity and needless to say a logical noun entity. Generally,
including is placed next to the noun entity it refers to. But that is not the case always.

This sentence is an example of that scenario. In this sentence, including is correctly modifying
numerous products. It is jumping over the that clause modifier that modifies the same entity. Now
that clauses are always placed next to the entity that they modify. So here, it is not possible to
change the placement of that clause. Since including is pretty flexible, it has been separated from
numerous products by adding a modifier in between. This usage is absolutely correct.

Take a look at the following correct official sentences:

1. A study by the Ocean Wildlife Campaign urged states to undertake a number of remedies to reverse
a decline in the shark population, including establishing size limits for shark catches, closing state
waters for shark fishing during pupping season, and requiring commercial fishers to have federal shark
permits. (GMAT Prep - Choice D).

including is jumping over the modifier to reverse population to modify remedies.

2. The new image of Stone Age people as systematic hunters of large animals, rather than mere
scavengers of meat, has emerged from the examination of tools found in Germany, including three
wooden spears that archaeologists believe to be about 400,000 years old. (GMAT Prep - Choice E)

including here is skipping the verb-ed modifier found in Germany to modify tools because that
modifier cannot be placed elsewhere.

3. Bluegrass musician Bill Monroe, whose repertory, views on musical collaboration, and vocal style
influenced generations of bluegrass artists, also inspired many musicians, including Elvis Presley and
Jerry Garcia, whose music differed significantly from his own. (GMAT Prep & Verbal 2nd Edition #107
Choice B)

Here including is preceded by a comma. Still it does not modify the preceding clause but the
preceding noun. SO here it is placed next to the entity it modifies.

Hi Shraddha
It seems there is an exception to both the verb-ing rules :
1. Placement after a clause not preceded by a comma
The rule states verb-ing modifier should modify the preceding nound or noun phrase but below example
shows the exception to this rule :
Exception : There are numerous products that are used to seal a bottle of wine including a cork, a
screwcap, and a newly released item called the Vino-Lok that combines the elegance of a cork with the
practicality of a screwcap.
Hi cu ny hi ngu
1. Placement after a clause preceded by a comma
- Either presents an additional information about the preceding clause or presents the result of
preceding clause
Exception : 3. Bluegrass musician Bill Monroe, whose repertory, views on musical collaboration, and
vocal style influenced generations of bluegrass artists, also inspired many musicians, including Elvis
Presley and Jerry Garcia, whose music differed significantly from his own. (GMAT Prep & Verbal 2nd
Edition #107 Choice B)

Need help to understand how do we differentiate b/w a right & a wrong choice for verb-ing modifier
with all the exceptions stated above for it.

Hi there,

Just add this exception to the rules including does not really follow the rules of verb-ing modifiers.
Whether it is separated by comma or not, it modifies the preceding noun (or noun + modifiers) only.
Think of including to play a role of "for example" (Again not in literal translation but only in a role).

So basically, the thing to keep in mind is that "including" is a versatile NOUN modifier. It always
modifies a NOUN. But it is usually separated by a comma. And it may modify a noun that is separated
by modifiers as well. Here are a few official sentences:
Example 1: (GMAT Prep)
A study by the Ocean Wildlife Campaign urged states to undertake a number of remedies to reverse a
decline in the shark population, which includes the establishment of size limits for shark catches,
closing state waters for shark fishing during pupping season, and requiring commercial fishers to have
federal shark permits.

D. including establishing size limits for shark catches, closing

Comma + Including modifies "REMEDIES". But look at the modifiers that lie between this modifier and
the modified noun.

Example 2: (GMAT Prep)


The new image of Stone Age people as systematic hunters of large animals, rather than merely
scavenging for meat, have emerged from the examination of tools found in Germany including , three
wooden spears that archaeologists believe to be about 400,000 years old.

E)mere scavengers of meat, has emerged from the examination of tools found in Germany, including

Comma + Including modifies "TOOLS". But look at the modifier that lie between this modifier and the
modified noun.

Example 3: (GMAT Prep & Verbal 2nd Edition #107)


Bluegrass musician Bill Monroe, whose repertory, views on musical collaboration, and vocal style were
influential on generations of bluegrass artists, was also an inspiration to many musicians, that included
Elvis Presley and Jerry Garcia, whose music differed significantly from his own.
Correct has , including where including modifies preceding noun

B) influenced generations of bluegrass artists, also inspired many musicians, including Elvis Presley and
Jerry Garcia, whose music differed significantly from
Comma + Including modifies "musicians" - in this case the closest noun.

Definition: A dangling modifier is a word or phrase that modifies a word not clearly
stated in the sentence. A modifier describes, clarifies, or gives more detail about a
concept.

Initial noun modifier lun modify cho subject ca cu ng ngay sau du phy. Khi thy
mt cu bt u vi mt initial modifier, cc bn cn check xem ci modifier c th
modify cho ci danh t ng sau du phy hay khng? Nu may mn bn c th loi 2-
3 cu sai vi li ny mt cch nh nhng khi loi p n bng li ny cc bn thm
ch cn khng cn c phn cn li ca cu, tit kim rt nhiu thi gian.

Chng ta s th phn tch qua mt vi v d:


V d 1:

Responding to growing demand for high-end vehicles, the interiors of the newest
models are so luxurious that they sell for nearly twice the price of last years models.

A. the interiors of the newest models are so luxurious that they sell
B. the interiors of the newest models are so luxurious that the cars are sold
C. auto makers have installed interiors in the newest models that are so luxurious
that they sell
D. the interior of the newest models are so luxurious that they are sold
E. auto makers have installed such luxurious interiors in the newest models that
these cars sell

Initial modifier ca cu ny l Responding to growing demand for high-end vehicles,


c xong ci modifier ny cc bn phi ngay lp tc hnh thnh trong u cu hi: ci
g c th lm subject cho response? ng nhin phi l mt ngi ri, tt c
nhng cu no khng start vi mt danh t ch ngi c th b loi m khng cn phi
ngh nhiu. Vi li ny chng ta c th loi b A, B, v D m khng cn care phn ng
sau ca cu vit g.

cu C c 2 vn : th nht that are so luxurious modify cho newest models, trong


khi y ng ra n phi modify cho interiors. Nu mun ni cu logic th phi l:
v bn makers lp luxurious interiors nn gi ca newest models t gp i gi models
ca nm trc. Th hai, chng ta khng xc nh c they refer ti danh t no
trong cu: auto makers, interiors, hay models?

p n chnh xc l E. Tuy nhin trong cu ny c mt cm cc bn cn phi ch :


these cars sell for . ng ra th these cars khng th sell ci g c, v cch vit
ny khng m ch these cars sell something. y l correct idiom trong GMAT, n c
ngha l newest models c bn vi gi gp i last years models.

Responding to growing demand for high-end vehicles, the interiors of the newest
models are so luxurious that they sell for nearly twice the price of last years models.

A. the interiors of the newest models are so luxurious that they sell
B. the interiors of the newest models are so luxurious that the cars are sold
C. auto makers have installed interiors in the newest models that are so
luxurious that they sell
D. the interior of the newest models are so luxurious that they are sold
E. auto makers have installed such luxurious interiors in the newest models that
these cars sell

V d 2:
Hailed as a key discovery in the science of evolution, the fossils of a large scaly
creature resembling both a fish and a land-animal provide evidence of a possible link in
the evolutionary chain from water-based to land-based organisms.

A. the fossils of a large scaly creature resembling both a fish and a land-animal
provide evidence of
B. a large scaly creature resembling both a fish and a land-animal provides fossils
that are evidence
C. a large scaly creature, whose fossils resemble both a fish and a land-animal,
provides evidence of
D. the fossils of a large scaly creature, which resembles both a fish and a land-
animal, provides evidence of
E. the fossils of a large scaly creature resemble both a fish and a land-animal and
provide evidence of

Cu ny start vi modifier Hailed as a key discovery in the science of evolution, check


th danh t sau du phy xem ci g c th l key discovery: the fossils hay a large
scaly creature? Ci c discover phi l the fossils, chng ta c th loi ngay B v
C.

Fossils l s nhiu do ng t phi chia theo theo s nhiu, tip tc loi D.

cu E, resemble v provide song song vi nhau v cng c subject l fossils, fossils


khng th resemble both a fish and a land-animal. Ci c th resemble l a large scaly
creature. Loi E.

p n chnh xc l A.

Hailed as a key discovery in the science of evolution, the fossils of a large scaly
creature resembling both a fish and a land-animal provide evidence of a possible link in
the evolutionary chain from water-based to land-based organisms.

A. the fossils of a large scaly creature resembling both a fish and a land-animal
provide evidence of
B. a large scaly creature resembling both a fish and a land-animal provides fossils
that are evidence
C. a large scaly creature, whose fossils resemble both a fish and a land-animal,
provides evidence of
D. the fossils of a large scaly creature, which resembles both a fish and a land-
animal, provides evidence of
E. the fossils of a large scaly creature resemble both a fish and a land-animal and
provide evidence of

V d 3:
Given its authoritative coverage of other science topics, the textbooks chapter on
genetics is surprisingly tentative, which leads one to doubt the authors scholarship in
that particular area.

A. the textbooks chapter on genetics is surprisingly tentative, which leads


B. the chapter of the textbook on genetics is surprisingly tentative, leading
C. the textbook contains a surprising and tentative chapter on genetics, which leads
D. the textbooks chapter on genetics is surprisingly tentative and leads
E. the textbook is surprisingly tentative in its chapter on genetics, leading

v d 1 v 2 vic nhn ra li modifier kh n gin, cu ny vic pht hin li kh


hn v da nhiu vo meaning.

Initial modifier ca cu ny l Given its authoritative coverage of other science topics,


sau du phy c 2 danh t the textbooks chapter on genetics v the textbook, vy
u s l danh t c modify? c qua th c 2 danh t u make sense, tuy nhin
cha kha y chnh l its authoritative coverage of other science topics. Mt cch
logic th the chapter on genetics s cover thng tin v genetics, cn thng tin v
other science topics s c covered other chapters, do subject s l the
textbook. Loi A, B, v D.

Cu C c 2 vn : (1) surprising v tentative cng modify cho chapter, ngha chapter


surprising ch khng phi tentativeness ca chapter surprising; (2) t which c
th modify cho either chapter or genetics, nhng c 2 ci u khng th l subject cho
ng t leads. Meaning ca cu ny l, v cun sch cha surprisingly tentative
chapter on genetics nn dn n vic ngi ta nghi ng kin thc ca tc gi lnh
vc .

p n chnh xc l E. Vic dng verbing modifier l rt chun, leading one to doubt


l consequence ca clause ng trc, v y chnh l ngha ca verbing modifier
nm cui cu.

Given its authoritative coverage of other science topics, the textbooks chapter on
genetics is surprisingly tentative, which leads one to doubt the authors scholarship in
that particular area.

A. the textbooks chapter on genetics is surprisingly tentative, which leads


B. the chapter of the textbook on genetics is surprisingly tentative, leading
C. the textbook contains a surprising and tentative chapter on genetics, which leads
D. the textbooks chapter on genetics is surprisingly tentative and leads
E. the textbook is surprisingly tentative in its chapter on genetics, leading
V d 4:

A mixture of reggae and jazz, the music of the artist was more innovative than most of
his contemporaries were.

A. the music of the artist was more innovative than most of his contemporaries were
B. the artist was more innovative in his music than most of his contemporaries
C. the artists music was more innovative than that of most of his contemporaries
D. the music of the artist being more innovative in comparison to the music of most
of his contemporaries
E. the artist, in his music, was more innovative than most of his contemporaries
were

Ci g c th l A mixture of reggae and jazz? Chng ta ko th mix reggae v jazz


to ra mt artist c, nh nhng loi B v E.

Chng ta ch c th compare music vi music ch khng th compare the music of the


artist vi his contemporaries c. Loi A.

Cui cng cu D khng c ng t nn khng phi l mt cu hon chnh.

p n chnh xc l C.

A mixture of reggae and jazz, the music of the artist was more innovative than most of
his contemporaries were.

A. the music of the artist was more innovative than most of his contemporaries were
B. the artist was more innovative in his music than most of his contemporaries
C. the artists music was more innovative than that of most of his contemporaries
D. the music of the artist being more innovative in comparison to the music of most
of his contemporaries
E. the artist, in his music, was more innovative than most of his contemporaries
were

V d 5:

Although closed for renovation, architecture students with a college pass can gain entry
to the historic monument.

A. Although closed for renovation


B. Although it is closed for renovation
C. Closed for renovation
D. Closed on account of renovation
E. Having closed for renovation

Trong cu ny phn initial modifier l phn c gch chn, subject ca cu ny l


architecture students, tuy nhin students khng th closed hoc b closed c. Ci b
closed y l the historic monument. Trong c 4 cu A, C, D, E th closed u l
modifier cho students do chng ta c th loi c 4 cu ny.

p n chnh xc l B. Trong cu ny pronoun it refers ti the historic monument


nn chng ta khng th hiu nhm l students b closed c.

V d 6: y l 2 cu khc nhau, 2 p n khc nhau (c 2 cu u l official)

In addition to her work on the Miocene hominid fossil record, Mary Leakey contributed
to archaeology with her discovery of the earliest direct evidence of hominid activity and
painstakingly documenting East African cave paintings.

A. Leakey contributed to archaeology with her discovery of the earliest direct


evidence of hominid activity and painstakingly documenting
B. Leakey contributed to archaeology by her discovery of the earliest direct
evidence of hominid activity and by painstakingly documenting
C. Leakey was a contributor to archaeology with her discovery of the earliest direct
evidence of hominid activity and with her painstaking documentation of
D. Leakeys contributions to archaeology include her discovery of the earliest direct
evidence of hominid activity and her painstaking documentation of
E. Leakeys contributions to archaeology include her discovering the earliest direct
evidence of hominid activity and painstaking documentation of

In addition to her work on the Miocene hominid fossil record, Mary Leakey contributed
to archaeology through her discovery of the earliest direct evidence of hominid activity
and through her painstaking documentation of East African cave paintings.

A. Mary Leakey contributed to archaeology through her discovery of the earliest


direct evidence of hominid activity and through her painstaking documentation of
B. Mary Leakey contributed to archaeology by her discovery of the earliest direct
evidence of hominid activity and painstakingly documenting
C. Mary Leakey was a contributor to archaeology by discovering the earliest direct
evidence of hominid activity and with her painstaking documentation of
D. Mary Leakeys contributions to archaeology include her discovery of the earliest
direct evidence of hominid activity and painstakingly documenting
E. Mary Leakeys contributions to archaeology include her discovering the earliest
direct evidence of hominid activity and painstaking documentation of

C 2 cu ny start vi modifier In addition to her work , vy subject ca cu ny s


l Mary Leakey hay Mary Leakeys contributions?
Nu cc bn search cc din n s tm c cc cu tr li kiu nh: v modifier l
In addition to her work, do subject ca cu ny cn phi l mt ci g parallel vi
her work, y s l her contributions. Trc y mnh tng ngh nh vy, v thm
ch c Ron Purewal cng tng ngh nh vy.

Tuy nhin cu tr li l c 2 u possible, ty theo meaning ca cu. hiu ti sao c


hai u c th lm ch ng ca cu, trc tin chng ta cn hiu In addition to her
work c vai tr ng php g trong cu. y l mt prepositional phrase, v v mt
ng php th mt prepositional phrase c th ng vai tr l mt adjective (modify cho
noun) hoc mt adverb (modify cho verb, adjective).

Trong c 2 cu ny th In addition to her work u c vai tr l adverb modify cho


2 verb include v contributed.

cu u chng ta c th hiu Mary Leakeys contributions include: her work on the


Miocene hominid fossil record, her discovery, and her documentation. cu th 2 Mary
Leaky contributed: her work on the Miocene hominid fossil record, her discovery, and
her documentation.

Ly v d n gin hn:

In addition to apple, Mary bought orange and grape.


In addition to apple, the bucket contains orange and grape.

Apple s parallel vi orange v grape bi v chng u c Mary mua hoc u nm


trong bucket.

Do li chnh trong cu ny l parallelism: her work, her discovery v her


documentation cn parallel vi nhau (painstaking l adjective). Khng th dng
discovering hay documenting c.

cu C, phrase In addition to her work khng c support g cho vic Mary Leakey
was a contributor.

p n chnh xc ca 2 cu ny l: D v A.

Note: correct idiom contribute to something through something, tuy nhin chng ta
khng cn da vo idiom lm cu ny, lun c nhng li khc bn cnh idiom.

n y chc cng cho Dangling Modifier ri. Hi vng khng bn no lm sai phn
ny.
Hi Folks,

Try this official question (OG Verbal 2#100) to aee if you have understood the concept discussed
in the article well.

Sixty-five million years ago, according to some scientists, an asteroid bigger than Mount Everest

slammed into North America, which, causing plant and animal extinctions, marks the end of the
geologic era known as the Cretaceous Period.
A. which, causing plant and animal extinctions, marks

B. which caused the plant and animal extinctions marking

C. and causing plant and animal extinctions that mark

D. an event that caused plant and animal extinctions, and it marks

E. an event that caused the plant and animal extinctions that mark

Thanks.
Shraddha

Hi debayan,
Yes, you chose the correct answer. However, I would love to see a little detailed analysis of how
you reach to the correct answer. Good job nonetheless.

Here is my analysis of the question:

Meaning

According to some scientists, 65 million years ago, an asteroid, bigger than Mount Everest,
slammed into North America.
This caused extinction of all the plants and animals.
So, logically it is the extinction of living entities that marks the end of the geologic era known as
the Cretaceous Period.
But the way this sentence is written, it seems that this event of a gigantic asteroid slamming into
North America marks the end of this particular geological era. This is not logical.

Error Analysis

which clause is modifying the event of an asteroid slamming the North America. Being a noun
modifier, which clause can only modify a noun. Modifier error.
Verb-ing modifier causing plant and animal extinctions is neither worded nor placed correctly
to suggest that slamming of the asteroid led to the extinction of plants and animals. Modifier error.
As discussed in the meaning analysis, it is not clear from the sentence that extinction marks the
end of the Cretaceous Period. Meaning error.

PoE

A. which, causing plant and animal extinctions, marks: Incorrect for the reasons stated above.

B. which caused the plant and animal extinctions marking: Incorrect. Same which clause error in
A.

C. and causing plant and animal extinctions that mark: Incorrect. There is no main verb after comma
+ and, leading to fragment.

D. an event that caused plant and animal extinctions, and it marks: Incorrect. which refers to
plural extinctions that needs plural verb mark. Singular verb marks is incorrect. Commented [a4]: It l pronoun khng th modify c action
planet hit Earth
E. an event that caused the plant and animal extinctions that mark: Correct..
A group of paleontologists recently announced that a site in Utah has yielded fossils of some of the
biggest armored dinosaurs ever found, and that they were at least 25 million years older than any
similar dinosaur type previously found in North America.

(A) and that they were at least 25 million years older than any similar dinosaur type previously
(B) and they are at least 25 million years older than those of any similar dinosaur type that previously
was
(C) and the fossils are at least 25 million years older than any similar dinosaur types that previously
were
(D) fossils that are at least 25 million years older than those of any similar dinosaur type previously
(E) fossils at least 25 million years older than similar dinosaur types previously

Here is the detailed solution of this one.

A group of paleontologists recently announced that a site in Utah has yielded fossils of some of the
biggest armored dinosaurs ever found, and that they were at least 25 million years older than any
similar dinosaur type previously found in North America.

Since the meaning of the sentence is pretty simple to understand, lets move on to error analysis.

Error Analysis:

1. The paleontologists found some fossils and on the basis of this discovery, they announced these
dinosaurs were at least 25 million years older than any other similar type previously found in North
America. Hence, these two actions are not independent of each other. There is a causal relationship
here. Hence, presenting these two information as two independent entities in a parallel list does not
make sense.

2. Although its a not a deterministic error, its worth mentioning. Comma + and is used to join more
than two entities in the list. Here it has been used to join two entities. Since comma lies in the non- Commented [a5]: ????
underlined portion of the sentence, we do need suitable structure.

3. Also pronoun they can refer to dinosaurs as well as fossils.

PoE:

(A) and that they were at least 25 million years older than any similar dinosaur type
previously: Incorrect for the reasons stated above.

(B) and they are at least 25 million years older than those of any similar dinosaur type that previously
was: Incorrect.
1. This choice repeats all the errors of Choice A.
2. those of now stands for fossils. This leads to incorrect comparison. Logically, the comparison
should be between two types of dinosaurs. But here the comparison is incorrect any way. If they
refers to fossils, then the comparison is between fossils. This is certainly incorrect. If they refers
to dinosaurs then the comparison is illogical because now dinosaurs are compared to fossils.

(C) and the fossils are at least 25 million years older than any similar dinosaur types that previously
were: Incorrect. Incorrect. This choice as illogical comparison. Entities compared fossils and any
similar dinosaur types.

(D) fossils that are at least 25 million years older than those of any similar dinosaur type
previously: Correct.

(E) fossils at least 25 million years older than similar dinosaur types previously: Incorrect. Same
comparison error as in choice C.

OFFICIAL EXAMPLE: GMAT Prep

So lets first understand the meaning of the sentence. The sentence says that one automobile
manufacturer has announced plans to increase the average fuel efficiency of its SUVs by 25% over the
next five years. This increase will amount to roughly five miles per gallon and will represent the first
significant change in the fuel efficiency of any class of passenger vehicle in almost 20 years.

Now the way this sentence uses the verb-ing modifiers amounting and resulting, both preceded by
comma, these modifiers seems to present the result of the preceding clause One automobile
manufacturer has announced plans. Here, both the modifiers are referring to the subject and the
verb of the preceding main clause. Logically, this does not make sense because announcement of plans
cannot amount to five miles per gallon. The act of mere announcement cannot even represent first
significant change. Hence, the use of verb-ing modifiers is incorrect in this sentence.

Now lets analyze the sentence with correct answer choice D:

In this sentence, verb-ing modifiers amounting and representing have replaced with noun +
noun modifiers an increase that would amount to and (an increase) that would represent. Here,

an increase = noun
that would amount and that would represent = noun modifiers (relative pronoun clauses)

By converting the verb-ing modifiers into noun + noun modifier, we are able to rectify the modification
error easily because now, unlike the verb-ing modifiers, the noun + noun modifier need not modify the
subject and the verb of the preceding clause. It can easily zoon into increase to present more
information about this aspect of the preceding clause. This is the beauty of such modifiers.

Now lets analyze the sentence with correct answer choice D:

In this sentence, verb-ing modifiers amounting and representing have replaced with noun +
noun modifiers an increase that would amount to and (an increase) that would represent. Here,

an increase = noun
that would amount and that would represent = noun modifiers (relative pronoun clauses)
By converting the verb-ing modifiers into noun + noun modifier, we are able to rectify the modification
error easily because now, unlike the verb-ing modifiers, the noun + noun modifier need not modify the
subject and the verb of the preceding clause. It can easily zoon into increase to present more
information about this aspect of the preceding clause. This is the beauty of such modifiers.

Prepositional phrases are very versatile modifiers. They modify nouns as well verbs. The modification
of prepositional phrases depends upon the context of the sentence and hence their placement in the
sentence.

Now lets take the OG sentence to understand the modification of the prepositional phrase.

Defense attorneys have occasionally argued that their clients misconduct stemmed from a reaction to
something ingested, but in attributing criminal or delinquent behavior to some food allergy, the
perpetrators are in effect told that they are not responsible for their actions.

The sentence says that sometimes to save their clients, defense attorneys argue that their client
committed the crime because of reaction of some food. If it is proved that the perpetrator is allergic to
some food and he had ingested that before committing the crime, then the perpetrators are not held
responsible for their misconduct.

The prepositional phrase in attributing criminal behavior modifies the subject of the following
clause the perpetrators. Why so? Because the prepositional phrase says in attributing. This word
attributing denotes an action that needs to be done by someone. Hence, here we are dealing with
a prepositional phrase that needs a does, a noun entity. So it modifies perpetrators. And this
modification does not make sense because the perpetrators do not attribute the criminal behavior to
some food allergy.

Hi @bagdbmba,

You're right: "the perpetrators are told" is in the passive voice, since they aren't doing the telling -
they're told by someone else.

"In effect" is a prepositional modifier; i.e., a modifier that begins with a preposition. Here, it functions
as an adverb, telling us more about the action "told". So, the meaning here is that in saying that the
perpetrator's misbehavior is the result of an allergic reaction, one is practically telling the perpetrators
that they aren't to blame for their misbehavior.
Lets take another official sentence here:

The nephew of Pliny the Elder wrote the only eyewitness account of the great eruption of Vesuvius in
two letters to the historian Tacitus.

Prepositional phrase in two letters, the way it is placed in this sentence, modifies the great
eruption of Vesuvius. But logically it should modify the verb wrote the only eyewitness account.
Change in the placement of this modifier makes this incorrect sentence correct.

In two letters to the historian Tacitus, the nephew of Pliny the Elder wrote the only eyewitness
account of the great eruption of Vesuvius.

The opening modifier now correctly modifies the following clause. Commented [a6]: Opeing modifier l ci g vy h tri

Using the stick, Tom killed the snake. Commented [a7]: Cu ny r rng l modifier c action kiiled
What does the modifier using the stick modify in the above sentence? the snake m sao bu l modify Tom l sao o hiu
lm????????????

As explained in the above article, when a verb-ing modifier is placed at the beginning of the sentence,
it modifies the subject of the sentence. It is because the verb-ing modifier denotes an action that
needs to be done by someone. So, the subject of the sentence becomes the doer of this action.
In the above example, Tom used the stick to kill the snake. So, Tom is the doer of the action denoted
by using.

In 1945, World War II ended.


What does the modifier in 1945 modify in the above sentence?

In this example, in 1945 is a prepositional phrase. Prepositional phrases are very versatile modifiers
and they can modify nouns as well as verbs. This modification depends upon the context of the
sentence.
In the above sentence, the prepositional phrase in 1945 modifies the verb ended.

Now, coming to your question:


In two letters to the historian Tacitus, the nephew of Pliny the Elder wrote the only eyewitness account
of the great eruption of Vesuvius.

The modifier in two letters to the historian Tacitus modifies the verb wrote in the given sentence. It
does not modify the subject.
TAKE AWAYS
1. Prepositional phrases can modify nouns as well as verbs.

2. Verb-ing modifier placed at the beginning of the sentence modifies the subject of the preceding
clause. Commented [a8]: ????????????? ko c modify cho action l sao

Scientists have recently discovered what could be the largest and oldest living organism on Earth, a Commented [a9]: A
giant fungus that is an interwoven filigree of mushrooms and root like tentacles spawned by a single
fertilized spore some 10,000 years ago and extending for more than 30 acres in the soil of a Michigan
forest.

A. extending
B. extends
C. extended
D. it extended
E. is extending

Scientists have recently discovered what could be the largest and oldest living organism on Earth, a
giant fungus that is an interwoven filigree of mushrooms and root like tentacles spawned by a single
fertilized spore some 10,000 years ago and extending for more than 30 acres in the soil of a Michigan
forest.

Now let us apply this test on the giant fungus. After cutting the flab, this is the core that we get:
a giant fungus spawned by a single fertilized pore some 10,000 years ago

So lets ask, did a giant fungus do the action of spawning? The answer is no because it is clearly
mentioned in the sentence that the action of spawning was done by a single fertilized pore. Hence,
the litmus test confirms that spawned is a verb-ed modifier.

Therefore, to maintain parallelism we need to use a form of extend that is used as a modifier. Note
that extended CAN be a verb-ed modifier. However, we need to find out if this form works in this
sentence as a modifier:

a giant fungus extended for more than 30 acres in the soil of a Michigan forest.

Did a giant fungus do the job of extension? Yes, it did. It is the giant fungus that extends for more
than 30 acres. This means that extended will work as simple past tense verb in this sentence and not
as a modifier. This is the reason why extended cannot be used since a verb CANNOT be parallel to a
verb-ed modifier.

Lets once again quickly look at the answer choices:


A. extending
B. extends
C. extended
D. it extended
E. is extending

A quick scan of these choices will tell you that all the answer choices B, C and E are verbs in different
forms while Choice E is a clause. Choice A extending is the only modifier that describes one of the
characteristics of a giant fungus. Hence, verb-ed modifier spawned and verb-ing modifier
extending are parallel entities because they both perform the same function, although they differ
structurally.

SAI NGU

39. According to scholars, the earliest writing was probably not a direct rendering of speech,
but was more likely to begin as a separate and distinct symbolic system of communication, and
only later merged with spoken language. (comparison problem)

(A) was more likely to begin as

(B) more than likely began as

(C) more than likely beginning from

(D) it was more than likely begun from

(E) it was more likely that it began

Trong cu ny, ch ng phn sau ca cu b mt i:

the earliest writing was probably not a direct rendering of speech, but more than likely the
earliest writing began as and merged...

V phn sau c and merged phi l began trong phn gch chn. A + C + D out!
E: it ambiguous B

40. In 1995 Richard Stallman, a well-known critic of the patent system, testied in Patent Ofce
hearings that, to test the system, a colleague of his had managed to win a patent for one of
Kirchhoffs laws, an observation about electric current rst made in 1845 and now included in
virtually every textbook of elementary physics.

(A) laws, an observation about electric current rst made in 1845 and
(B) laws, which was an observation about electric current rst made in 1845 and it is

(C) laws, namely, it was an observation about electric current rst made in 1845 and

(D) laws, an observation about electric current rst made in 1845, it is

(E) laws that was an observation about electric current, rst made in 1845, and is

APPOSITVE MODIFIER A

B: and it is run on sentence (no comma presented) + was should be were

C: namely? Also, a run on sentence

D: run on sentence

E: sounds like current was made in 1845; that refers to laws was is wrong

42. The Supreme Court has ruled that public universities can collect student activity fees even
with students objections to particular activities, so long as the groups they give money to will
be chosen without regard to their views.

(A) with students objections to particular activities, so long as the groups they give money to
will be

(B) if they have objections to particular activities and the groups that are given the money are

(C) if they object to particular activities, but the groups that the money is given to have to be

(D) from students who object to particular activities, so long as the groups given money are

(E) though students have an objection to particular activities, but the groups that are given the

money be

https://e-gmat.com/articles/?p=1207

A: they their must share 1 antecedent illogical meaning

B+C: fees have objections? ridiculous; C: to at the end of the modifier really? Crazy

4. Prep exam 1: The positive integer k has exactly two positive prime factors, 3 and 7. If k has
a total of 6 positive factors, including 1 and k, what is the value of K?
(1) 3^2 is a factor of k
(2) 7^2 is NOT a factor of k
K = 3x.7y since K has exactly 6 positive factors, (x+1).(y+1) = 6 K = 32.7 or K=3.72
each statement is sufficient.

Eg: find the number of factors of 420? 420=22.3.5.7 (420) = (2+1).(1+1).(1+1).(1+1) =


24 factors

12. prep exam 1: Of the students who eat in a certain cafeteria, each student either likes or
dislikes lima beans and each student either likes or dislikes brussels sprouts. Of these students,
2/3 dislike lima beans; and of those who dislike lima beans, 3/5 also dislike brussels sprouts.
How many of the students like Brussels sprouts but dislike lima beans?
(1) 120 students eat in the cafeteria
(2) 40 of the students like lima beans

Each statement is sufficient

33. Preparation exam 1: Is m+z > 0?


(1) m-3z > 0
(2) 4z-m > 0

(1) not enough

(2) not enough

(1)&(2) m-3z+4z-m>0 z>0. (1) m>3z>0 m+z>0 sufficient.

Preparation exam 1: Based on records from ancient Athens, each year young Athenian women
collaborated to weave a new woolen robe that they used to dress a statue of the goddess Athena
and that this robe depicted scenes of a battle between Zeus, Athena's father, and giants.

A. Based on records from ancient Athens, each year young Athenian women collaborated to weave a
new woolen robe that they used to dress

B. Based on records from ancient Athens, young Athenian women had collaborated to weave a new
woolen robe with which to dress

C. According to records from ancient Athens, each year young Athenian women collaborated to
weave a new woolen robe that they used to dress

D. Records from ancient Athens indicate that each year young Athenian women collaborated to
weave a new woolen robe with which they dressed
E. Records from ancient Athens indicate each year young Athenian women had collaborated to
weave a new woolen robe for dressing

A + B: based on modify women l khng ph hp. nhng ph n ny khng th


no da vo records.. c. --> A&B out.
C: robe that they used to dress h dung tm o chong ny thc hin vic mc o.
Hn na, d c ra v // bng cu trc: that X and that Y nhng trong cu ny, that u
tin m t 1 c tnh ca the robe, cu sau li m t mt s thc khc hiu: the robe
that they used to dress + the robe that this robe depicted
E: (&B) had collaborated th qkht khng ph hp din t 1 hnh ng lp i lp li
trong qu kh.
D: indicate that young women abc and that this robe xyz
Preparation exam 1: In archaeology, there must be a balance between explanation of the value and
workings of archaeology, revealing the mysteries of past and present cultures, and to
promote respect for archaeological sites.

A. between explanation of the value and workings of archaeology, revealing the mysteries of past
and present cultures, and to promote

B. among explaining the value and workings of archaeology, revealing the mysteries of past and
present cultures, and promoting

C. between explaining the value and workings of archaeology, revealing the mysteries of past and
present cultures, and when promoting

D. among explaining the value and workings of archaeology, the revelation of the mysteries of past
and present cultures, and to promote

E. between explaining archaeologys value and workings, in the revealing the mysteries of past and
present cultures, and in promoting

1st split: among v.s between look back at the meaning: balance has 3 laterals: explaining,
revealing., and promoting among.
A + C + E: out
D: explaining, the revelation, to promote: not parallel.
B: perfect.

HOW TO SEE THE BIG PICTURE? https://www.manhattanprep.com/gmat/blog/2015/08/17/how-to-


tackle-every-single-gmat-problem-seriously-part-3-2/ GO TO THIS LINK AND READ.
First, Glance at the problem. What does that even mean for SC? Theres so much text!

Dont read the sentence yet. Heres where to glance:

Start with the word before the underline and the first underlined word
Then glance down the first word themselves or two of each answer choice

Some words are markers all by: if you see the word and, you know something is going on with
parallelism. Other clues make themselves known when you compare differences in the
answers, and theres always one difference at the beginning of each answer choice. You may
not always know what the difference signifies, but if you train well, you can learn to spot clues
on a First Glance something like 70% to 80% of the time.
Preparation exam 1: As the honeybees stinger is heavily barbed, staying where it is inserted, this
results in the act of stinging causing the bee to sustain a fatal injury.

A. As the honeybees stinger is heavily barbed, staying where it is inserted, this results in the act of
stinging causing
B. As the heavily barbed stinger of the honeybee stays where it is inserted, with the result that the
act of stinging causes
C. The honeybees stinger, heavily barbed and staying where it is inserted, results in the fact that
the act of stinging causes
D. The heavily barbed stinger of the honeybee stays where it is inserted, and results in the act of
stinging causing
E. The honeybees stinger is heavily barbed and stays where it is inserted, with the result that the
act of stinging causes

X Result in Y only correct if X is an ACTION A + C + D are out.

A: This has no reference

Hn na, cu trc prep + Noun + V_ing ch ng khi prep refer ti Noun. L do: khi Prep + N +
V-ing th V-ing ng vai tr modifier k c thiu cm ny th cu vn make sense

The stinger (?) results in the act of stinging nonsense


I have a photo of my cousin playing football ok v nu b c cm playing football cng vn ok

B: As u cu khin cu tr thnh run-on out


Between 1990 and 2000 the global economy grew more than it did during the 10,000 years from the
beginning of agriculture to 1950. (comparison problem)
A. Between 1990 and 2000 the global economy grew more than it did during the 10,000 years from
the beginning of agriculture

B. Between 1990 and 2000 the global economy grew more than that during 10,000 years, from
when agriculture began

C. The growth of the global economy between 1990 and 2000 exceeds that which has been for
10,000 years from the beginning of agriculture

D. The growth of the global economy between 1990 and 2000 exceeds what it has been for 10,000
years, from when agriculture began

E. The growth of the global economy between 1990 and 2000 exceeded what it did for 10,000 years
from the beginning of agriculture

B: from when? C + D: has been khng ph hp t hin tng xy ra trc thi im 1950

E: the growth exceeded what it did: sai! Phi l what it was v the growth did what? It did nothing,
silly. Also, it trong D&E: ambiguous A is correct

The government of country X is more stable than that of country Y.


Can't use "it" (which would have to refer to the government of country X).
Hence "that", which is a tool specifically engineered to refer to "the government" WITHOUT the
modifier. That's the whole reason why it exists.

Tht ra mnh ln tn trong cu A l v n so snh mt cch m h: the economy grew more than it
did more g? ang so snh trn tiu ch g? Faster hay more beautifully??? FUCK you GMAC!

This time, Ron is on my side:

In fact, I think the usage of "more" without a following noun, adjective, or adverb will be highly
suspicious in most circumstances.
In other words, if you see something like "X is more than Y", versus alternatives such as "X
is higher than Y" or "X is greater than Y", you should in general be able to trust the latter.

Read more about comparing things in GMAT here

http://www.manhattanprep.com/gmat/blog/2012/10/25/comparing-things-in-gmatprep-sentence-
correction-2/
Most of the purported health benefits of tea comes from antioxidants - compounds also found
in beta carotene, vitamin E, and vitamin C that inhibit the formation of plaque along the
bodys blood vessels.

A. comes from antioxidants - compounds also found in beta carotene, vitamin E, and vitamin C that
B. comes from antioxidants - compounds that are also found in beta carotene, vitamin E, and
vitamin C, and they
C. come from antioxidants - compounds also found in beta carotene, vitamin E, and vitamin C, and
D. come from antioxidants - compounds that are also found in beta carotene, vitamin E, and vitamin
C and that
E. come from antioxidants - compounds also found in beta carotene, vitamin E, and vitamin C, and
they

YOU ARE FUCKING SILLY, TOAN!

1st split: look at the 1st underlined word come/comes

A: comes

B: comes C + D + E left. Since they listed out many factors check parallel!

C: Compounds also found in X, Y, Z, and inhibit (v) WRONG!

D: compounds that are found in XYZ, and that inhibit parallel!

E: Compounds also found and they not parallel. Also, after a dash (-), no independent clause

The single-family house constructed by the Yana, a Native American people who lived in what is
now northern california, was conical in shape, its framework of poles overlaid with slabs of bark,
either cedar or pine, and banked with dirt to a height of three to four feet.

a. banked with dirt to a height of


b. banked with dirt as high as that of
c. banked them with dirt to a height of
d. was banked with dirt as high as
e. was banked with dirt as high as that of

as high as khng i vi mt khong gi tr m i vi 1 gi tr c nh: the pole is as high as 3ft


ok

p n: A
Ban u mnh ln tn ti sao li l du (,) ch khng phi (;) tch 2 cu c c lp. Nhng
thc cht c on t its ti ht cu, ch l appositive modifier (absolute modifier) KINH KHNG.
Eg: She frowns upon him, her arms crossed

Eg: Roberta listened to her daughter, tears forming in her eyes

The health benefits of tea have been the subject of much research; in addition to its possibilities for
preventing and inhibiting some forms of cancer, the brewed leaves of Camellia sinensis may
also play a role in reducing the risk of heart disease and stroke.

A. in addition to its possibilities for preventing and inhibiting


B. in addition to its possibilities to prevent or inhibit
C. besides the possibility that it prevents and inhibits
D. besides the possible preventing and inhibiting of
E. besides possibly preventing or inhibiting

Sau phn gch chn l m t the brewed leaves of camellia.. Plural it, its b loi T_T

A+B+C ra i.

Possibility: ch c 2 cch dng: ~ of Noun v ~ that Noun verb A+B dng sai

D: and khng ph hp, v m t V-ing th possibly tt hn

E: correct

Neither nor 2 things. If more than 2:


Neither nor... nor nor

I like neither Japan nor Korea nor China.


OG - verbal review

In 1979 lack of rain reduced India's rice production to about 41 million tons, nearly 25 percent less
than those of the 1978 harvest. (comparison problem)

(A) less than those of the 1978 harvest


(B) less than the 1978 harvest
(C) less than 1978
(D) fewer than 1978
(E) fewer than that of India's 1978 harvest

Ngm hiu rng ang so snh 1979 harvest v 1978 harvest

A: refer to 41mil tons redundant C+D: 1978 ch l nm, khng so snh c vi 1979 harvest

D + E: fewer khng ph hp so snh harvest. Khng th so snh tons y c

Only seven people this century have been killed by the great white shark, the man-eater of the
movies - less than those killed by bee stings. (comparison problem)

a. movies--less than those


b. movies--fewer than have been
c. movies, which is less than those
d. movies, a number lower than the people
e. movies, fewer than the ones

7 people fewer than mi chnh xc A + C: oute

D: so snh a number vi the people?!!?! lower than the number of people who have been killed
by bee stings th ok

E: the ones khng c precedent + have been killed khng // vi killed

Rising inventories, when unaccompanied correspondingly by increases in sales, can lead to


production cutbacks that would hamper economic growth.

(A) when unaccompanied correspondingly by increases in sales, can lead


(B) when not accompanied by corresponding increases in sales, possibly leads
(C) when they were unaccompanied by corresponding sales increases, can lead
(D) if not accompanied by correspondingly increased sales, possibly leads
(E) if not accompanied by corresponding increases in sales, can lead

A: when ko ph hp, unaccompanied at best awkward, correspondingly: ambiguous


B: inventoriesleads S-V agreement was broken

C: were can lead verb tense

D: ging B

E: correct

18. Warning that computers in the United States are not secure, the National Academy of Sciences
has urged the nation to revamp computer security procedures, institute new emergency response
teams, creating a special nongovernment organization to take charge of computer security planning.

(A) creating a special nongovernment organization to take

(B) creating a special nongovernment organization that takes

(C) creating a special nongovernment organization for taking

(D) and create a special nongovernment organization for taking

(E) and create a special nongovernment organization to take

A: Urged the nation to revamp, institue., expected: and + V A+B+C: Out!


Between E&D: D is awkward

28. In virtually all types of tissue in every animal species dioxin induces the production of enzymes
that are the organisms trying to metabolize, or render harmless, the chemical that is irritating it.

(A) trying to metabolize, or render harmless, the chemical that is irritating it.
(B) trying that it metabolize, or render harmless, the chemical irritant
(C) attempt to try to metabolize, or render harmless, such a chemical irritant
(D) attempt to try and metabolize, or render harmless, the chemical irritating it
(E) attempt to metabolize, or render harmless, the chemical irritant

In A: it is ambiguous

B: it singular, but metabolize, render are in plural form

C+D: redundant

E: correct

** Irritant: (n) cht kch ng, cht kch thch

29. Emily Dickinsons letters to Susan Huntington Dickinson were written over a period beginning a
few years before Susans marriage to Emilys brother and ending shortly before Emilys death in
1886, outnumbering her letters to anyone else.
(A) Dickinson were written over a period beginning a few years before Susans marriage to Emilys
brother and ending shortly before Emilys death in 1886, outnumbering

(B) Dickinson were written over a period that begins a few years before Susans marriage to Emilys
brother and ended shortly before Emilys death in 1886, outnumber

(C) Dickinson, written over a period beginning a few years before Susans marriage to Emilys
brother and that ends shortly before Emilys death in 1886 and outnumbering

(D) Dickinson, which were written over a period beginning a few years before Susans marriage to
Emilys brother, ending shortly before Emilys death in 1886, and outnumbering

(E) Dickinson, which were written over a period beginning a few years before Susans marriage to
Emilys brother and ending shortly before Emilys death in 1886, outnumber

B: begins 1000 years ago??

C+D: fragment since there is no verb for the subject letters

A: grammatically correct, but weird meaning because outnumbering here implies some kinda
causal relationship (showing the result).

Eg: My brother, who ate a lot of bagels, graduated last year 2 independent observations

My brother ate a lot of bagels, graduating last year implies that he graduated because he ate the
bagels

Remember, COMA + V-ing present 1 of 2 functions: (1) the RESULTS of the action; (2)
describes HOW the action was conducted. In this eg, this structure serves non of the above 2.

E: correct

42 To develop more accurate population forecasts, demographers have to know a great deal more
than now about the social and economic determinants of fertility. (comparison problem)

(A) have to know a great deal more than now about the social and economic

(B) have to know a great deal more than they do now about the social and economical

(C) would have to know a great deal more than they do now about the social and economical

(D) would have to know a great deal more than they do now about the social and economic

(E) would have to know a great deal more than now about the social and economic

Economic: "having to do with "economics and/or the economy". (this is the intended meaning.)
"economical" means "efficient" or "at low cost/expense". (this is not the intended meaning.)

B+C out.
I have to study harder than now compare the intense of study vs. NOW? should be: I have
to study harder than I do now

A+E: out

Ch cn li ring em, D

Lu : trong vic so snh, c th omit s ng t v sau, khi ng t mun nhc li CHNH XC


l ng t ban u nhc ti, vd: I have to know more than he (knows/does) nhng trong cu
trn: the first verb is in a hypothetical/conditional tense (would have to know...), while the second
verb is in the present tense (they do [= know] now). you are not allowed to imply such tense
transitions.
if you omit the verb, the omitted verb will be assumed to be in the same tense as its parallel
counterpart. since you have to change tenses in this problem, you must include both verbs.

50: According to some analysts, the gains in the stock market reflect growing confidence that the
economy will avoid the recession that many had feared earlier in the year and instead come in for a
"soft landing", followed by a gradual increase in the business acitivity.

A) that the economy will avoid the recession that many had feared earlier in the year and instead
come

B) In the economy's ability to avoid the recession, something earlier in the year many had feared,
and instead to come

C) In the economy to avoid the recession, what many feared earlier in the year, rather to come

D) In the economy to avoid the recession many were fearing earlier in the year, and rather to come

E) that the economy will avoid the recession that was feared earlier this year by many, with it
instead coming

B+C+D: confidence in X to do Y: unidiomatic, ( C): what khng ph hp thay that, (D): were
fearing?

E: awkward, with it instead coming: unidiomatic

58. That educators have not anticipated the impact of microcomputer technology can hardly be said
that it is their fault : Alvin Tofer, one of the most prominent students of the future, did not even
mention microcomputers in Future Shock, published in 1970.

(A) That educators have not anticipated the impact of microcomputer technology can hardly be said
that it is their fault
(B) That educators have not anticipated the impact of microcomputer technology can hardly be said
to be at fault

(C) It can hardly be said that it is the fault of educators who have not anticipated the impact of
microcomputer technology

(D) It can hardly be said that educators are at fault for not anticipating the impact of
microcomputer technology

(E) The fact that educators are at fault for not anticipating the impact of microcomputer technology
can hardly be said

E: loi khng cn ngh v n c ngc li hon ton

A: it khng refer ti u c

B: at fault y khng ph hp, their fault s hp l hn. Be at fault c ngha l deserving


blame mt s vic khng th no at fault c. Ngn!

C: it th 2 trong cu khng refer ti u. It c s dng lm place holder ch trong trng hp n


ng u cu, cn khi n ng gia cu, li cn i sau 1 thng it na 2 thng phi cng l 1
th, m ci th th li ch bit l g???

D: awkward nhng li ko dnh li lm g, thi tm chp nhn. thng cht lm vua x m

60. The Olympic Games helped to keep peace among the pugnacious states of the Greek world in
that a sacred truce was proclaimed during the festivals month.

(A) world in that a sacred truce was proclaimed during the festivals month

(B) world, proclaiming a sacred truce during the festivals month

(C) world when they proclaimed a sacred truce for the festival month

(D) world, for a sacred truce was proclaimed during the month of the festival

(E) world by proclamation of a sacred truce that was for the month of the festival

For = because

B: proclaiming S l games ko th proclaim

A: festivals month: khng ph hp

C: when hm ti thi im sau khi proclaim xong, cc bn khng ha bnh na? distort

E: toooo wordy
67. The 32 species that make up the dolphin family are closely related to whales and in fact include
the animal known as the killer whale, which can grow to be 30 feet long and is famous for its
aggressive hunting pods.

(A) include the animal known as the killer whale, which can grow to be 30 feet long and is

(B) include the animal known as the killer whale, growing as big as 30 feet long and

(C) include the animal known as the killer whale, growing up to 30 feet long and being

(D) includes the animal known as the killer whale, which can grow as big as 30 feet long and is

(E) includes the animal known as the killer whale, which can grow to be 30 feet long and it is

1st split E+D: out (S: 32 species)

B + D: as big as long redundant. Ch cn ni big c no hoc long c no.

BIG NOTE: V-ing modifiers modify the entire clause. hiu l c h nh n (32 dng) u to ti
30ft, do n c lin quan mt thit ti c voi. distort the meaning. B+C out

Hn na, cch so snh: as big as 30ft long khng ch redundant m n cn hm cha mt li khc:
so snh sai. Khi ni con c growing as big as th ta expect phi l as big as something that
has the same size, vd: the whale, growing as big as an elephant, ch 30ft khng c ln, ko c
chiu di so snh.

The cow is as big as a small car OK; The cow is as big as the size of a small car wrong!

I run faster than he (does) OK; I run faster than his speed dead wrong!

I am taller than she is OK; I am taller then the height of hers/her height wrong!

LU : V-ING S B NGHA CHO C ON TRC, CH KHNG CH 1 DANH T!, tr


including: prep, khng phi V-ing m t noun ngay ng trc

69. Affording strategic proximity to the Strait of Gibraltar, Morocco was also of interest to the
French throughout the rst half of the twentieth century because they assumed that if they did not
hold it, their grip on Algeria was always insecure.

(A) if they did not hold it, their grip on Algeria was always insecure

(B) without it their grip on Algeria would never be secure

(C) their grip on Algeria was not ever secure if they did not hold it

(D) without that, they could never be secure about their grip on Algeria

(E) never would their grip on Algeria be secure if they did not hold it

A+C: if they did X, Y was insecure wrong. Must be: if they did X, Y would be
D: that cannot be a stand-alone pronoun, except for parallism: that of

The meaning of the sentence is that () the grib itself would not be secure, not that some countries
THINK/FEEL that the grib is not secure D distorts the meaning

E: inversion (never would their grib be) can only place at the beginning of the sentence, not at the
middle. It at the end of the sentence is ambiguous.

B: Im fucked up with you, though I can hardly satisfy with it

BIG NOTE: The French = the English = The Americans = The Odenio: plural!

75. More and more in recent years, cities are stressing the arts as a means to greater economic
development and investing millions of dollars in cultural activities, despite strained municipal
budgets and fading federal support.
(A) to greater economic development and investing
(B) to greater development economically and investing
(C) of greater economic development and invest
(D) of greater development economically and invest
(E) for greater economic development and the investment of

Cities are stressing and investing C+D+E: out!

Development economically wrong placement of adv B out

A: perfectly fine.
Note: Toan, for the last time, economic: related to trading, economy. Economical: efficient, save
money!
76. Combining enormous physical strength with higher intelligence, the Neanderthals appear as
equipped for facing any obstacle the environment could put in their path, but their relatively sudden
disappearance during the Paleolithic era indicates that an inability to adapt to some environmental
change led to their extinction.

(A) appear as equipped for facing any obstacle the environment could put in their path,

(B) appear to have been equipped to face any obstacle the environment could put in their path,

(C) appear as equipped to face any obstacle the environment could put in their paths,

(D) appeared as equipped to face any obstacle the environment could put in their paths,

(E) appeared to have been equipped for facing any obstacle the environment could put in their
path,

1. Appear as adj: means that they still appear this way (they disappeared long time ago, dear)

2. Equipped for doing: wrong. must be: equipped to do B for the answer!

1st split: appear vs. appeared. Since latter in the sentence, the author use present tense: indicates
appear should be better. D+E out

path should be in singular, since this word is just a metaphor. C+D out

@TOAN: you are caught between A&B. but you chose the weird A because in B, appear to have
been sounds like the Neanderthals still have something to do with present. BUT NO! after appear
or seem, the clause switch to the view of people at the tense of appear

they appear to have been: ok; they appered to have been: still very ok

i.e., it's not "have been equipped" (wrong in this problem); it's "appear TO HAVE been equipped".

in any case, you should probably just memorize this construction as a one-off idiomatic structure. if
you say "they seemed/appeared to...", then, no matter how remote the event is (in time), you use
this construction.

for instance:
it seems that the students cheated on the exam
(normal past tense in this construction)
but...
the students seem to have cheated on the exam
(not here)
in any case, though, you DO have to use an infinitive after "appear" or "seem" in this sort of
construction. and if you think about it, this is as past-tense as an infinitive can get. so that's why
you have to use it. To sum up:
"To ___" and "to have ___ed" are not verbs, so they don't have "tenses". Only verbs have tenses.

In the construction "You appear to ___", the only verb tense is the present tense of "appear".
77. A 1972 agreement between Canada and the United States reduced the amount of phosphates
that municipalities had been allowed to dump into the Great Lakes.

(A) reduced the amount of phosphates that municipalities had been allowed to dump

(B) reduced the phosphate amount that municipalities had been dumping

(C) reduces the phosphate amount municipalities have been allowed to dump

(D) reduced the amount of phosphates that municipalities are allowed to dump

(E) reduces the amount of phosphates allowed for dumping by municipalities

this is something that happened at one specific time in history. Before 1972, the level was X. At
some point in 1972whenever the agreement went into effectthe level was lowered to Y, and it
has stayed at Y ever since.

the level was lowered / was reduced / decreased / was brought down at a single point in time.
so, past tense. C+E: out

A + B: you cannot change the thing that was in the past out

E: allowed for dumping by companies: weird

79. Ryunosuke Akutagawas knowledge of the literatures of Europe, China, and that of Japan were
instrumental in his development as a writer, informing his literary style as much as the content of his
ction.

(A) that of Japan were instrumental in his development as a writer, informing his literary style as
much as

(B) that of Japan was instrumental in his development as a writer, and it informed both his literary
style as well as

(C)Japan was instrumental in his development as a writer, informing both his literary style and

(D)Japan was instrumental in his development as a writer, as it informed his literary style as much as

(E)Japan were instrumental in his development as a writer, informing both his literary style in
addition to

A+B: not //: Eu, China, and Japan makes much more sense out

E: knowledge is singular out

D: it: refers to development or knowledge? both cases are nonsense, by the way.

as much as requires // but his literary vs. the content of his fiction are not //

Also: development & information are functions, not cause and effect as (~because) distorts the
meaning.
80. The only way for growers to salvage frozen citrus is to process them quickly into juice
concentrate before they rot when warmer weather returns.

(A) to process them quickly into juice concentrate before they rot when warmer weather returns

(B) if they are quickly processed into juice concentrate before warmer weather returns to rot them

(C) for them to be processed quickly into juice concentrate before the fruit rots when warmer

weather returns

(D) if the fruit is quickly processed into juice concentrate before they rot when warmer weather
returns

(E) to have it quickly processed into juice concentrate before warmer weather returns and rots the
fruit

Citrus is singular!!!!!!!! A+B+C+D: out

C+D: if is nonsense out!

81. Fossils of the arm of a sloth found in Puerto Rico in 1991, and dated at 34 million years old,
made it the earliest known mammal of the Greater Antilles Islands.

(A) sloth found in Puerto Rico in 1991, and dated at 34 million years old, made it the earliest known
mammal of

(B) sloth, that they found in Puerto Rico in 1991, has been dated at 34 million years old, thus aking

it the earliest mammal known on

(C) sloth that was found in Puerto Rico in 1991, was dated at 34 million years old, making this the
earliest known mammal of

(D) sloth, found in Puerto Rico in 1991, have been dated at 34 million years old, making the sloth
the earliest known mammal on

(E) sloth which, found in Puerto Rico in 1991, was dated at 34 million years old, made the sloth the
earliest known mammal of

A: the fossils did not make the sloth the earliest mammal, but the dating did. it is ambiguous

B: They has no antecedent, fossils plural

C+E: same as B
89. Salt deposits and moisture threaten to destroy the Mohenjo-Daro excavation in Pakistan, the
site of an ancient civilization that ourished at the same time as the civilizations in the Nile Delta
and the river valleys of the Tigris and Euphrates.

(A) that ourished at the same time as the civilizations

(B) that had ourished at the same time as had the civilizations

(C) that ourished at the same time those had

(D) ourishing at the same time as those did

(E) ourishing at the same time as those were

C+D+E: out for those

B: out for had+Vpp. There is no time point in this sentence. A is correct.

NOTE: the V in .. at the same time as (did/flourished) the civilizations.. was omited

In comparing things, as is used when after it is a clause; like is used when after it is a N

Eg: as her brother, Alice aced the test wrong (unless Alice is her brother??!?!!)

Like her brother, Alice aced the test perfect!

90. The results of the companys cost-cutting measures are evident in its prots, which increased 5
percent during the rst 3 months of this year after it fell over the last two years.

(A) which increased 5 percent during the rst 3 months of this year after it fell

(B) which had increased 5 percent during the rst 3 months of this year after it had fallen

(C) which have increased 5 percent during the rst 3 months of this year after falling

(D) with a 5 percent increase during the rst 3 months of this year after falling

(E) with a 5 percent increase during the rst 3 months of this year after having fallen

A+B: it has no antecedent.

D+E: with an increase..: unclear. The profits increased, not the profits with an increase

E: having fallen implies that the profits never stop falling, which is wrong.
91. In an effort to reduce their inventories, Italian vintners have cut prices; their wines have been
priced to sell, and they are.

(A) have been priced to sell, and they are

(B) are priced to sell, and they have

(C) are priced to sell, and they do

(D) are being priced to sell, and have

(E) had been priced to sell, and they have

The word sell was omitted in the latter part of the clause they do sell: ok

A: the are (sell) wrong

B+D+E: they have (sell) wrong.

Note: only omit verb when 2 parts share the exact same verb.

94. Heating-oil prices are expected to be higher this year than last because reners are paying
about $5 a barrel more for crude oil than they were last year. (comparison problem)

(A) Heating-oil prices are expected to be higher this year than last because reners are paying
about $5 a barrel more for crude oil than they were

(B) Heating-oil prices are expected to rise higher this year over last because reners pay about $5 a
barrel for crude oil more than they did

(C) Expectations are for heating-oil prices to be higher this year than last years because reners
are paying about $5 a barrel for crude oil more than they did

(D) It is the expectation that heating-oil prices will be higher for this year over last because reners
are paying about $5 a barrel more for crude oil now than what they were

(E) It is expected that heating-oil prices will rise higher this year than last years because reners
pay about $5 a barrel for crude oil more than they did

X Higher than Y B+D: out

C: expectation are for: awkward; last years not // with this year

E: same as C correct: A
96. Even though Clovis points, spear points with longitudinal grooves chipped onto their faces, have
been found all over North America, they are named for the New Mexico site where they were rst
discovered in 1932.

(A) Even though Clovis points, spear points with longitudinal grooves chipped onto their faces, have
been found all over North America, they are named for the New Mexico site where they were rst
discovered in 1932.

(B) Although named for the New Mexico site where rst discovered in 1932, Clovis points are spear
points of longitudinal grooves chipped onto their faces and have been found all over North America.

(C) Named for the New Mexico site where they have been rst discovered in 1932, Clovis points,
spear points of longitudinal grooves chipped onto the faces, have been found all over North
America.

(D) Spear points with longitudinal grooves that are chipped onto the faces, Clovis points, even
though named for the New Mexico site where rst discovered in 1932, but were found all over North
America.

(E) While Clovis points are spear points whose faces have longitudinal grooves chipped into them,
they have been found all over North America, and named for the New Mexico site where they have
been rst discovered in 1932.

Although/ even though/ while: contrast markers construction: ~ + S+V, S2+V2

B+D: although named out

Also in D: even though but.. redundant

Have been found implies that the action still continues in the present nonsense C+E:out

97. Some anthropologists believe that the genetic homogeneity evident in the worlds people is the
result of a population bottleneckat some time in the past our ancestors suffered an event,
greatly reducing their numbers and thus our genetic variation.

(A) at some time in the past our ancestors suffered an event, greatly reducing their numbers

(B) that at some time in the past our ancestors suffered an event that greatly reduced their
numbers

(C) that some time in the past our ancestors suffered an event so that their numbers were greatly
reduced,

(D) some time in the past our ancestors suffered an event from which their numbers were greatly
reduced

(E) some time in the past, that our ancestors suffered an event so as to reduce their numbers
greatly,
A hyphen (-) is not used to connect 2 independent clauses A+D out

Also in A: reducing makes us think that our ancestors reduced their numbers (insane). Same as C
and E out.
D: not // with the latter part of the sentence

98. Ranked as one of the most important of Europes young playwrights, Franz Xaver Kroetz has
written 40 plays; his workstranslated into more than 30 languagesare produced more often
than any contemporary German dramatist.

(A) than any

(B) than any other

(C) than are any

(D) than those of any other

(E) as are those of any

STUPID mistake, Toan!

A+B+C: fatal mistake here is to compare works and people (dramatists) out!
E: more must be followed by than, not as

104. Beatrix Potter, in her book illustrations, carefully coordinating them with her narratives,
capitalized on her keen observation and love of the natural world.

(A) Beatrix Potter, in her book illustrations, carefully coordinating them with her narratives,

(B) In her book illustrations, carefully coordinating them with her narratives, Beatrix Potter

(C) In her book illustrations, which she carefully coordinated with her narratives, Beatrix Potter

(D) Carefully coordinated with her narratives, Beatrix Potter, in her book illustrations

(E) Beatrix Potter, in her book illustrations, carefully coordinated them with her narratives and

A: not //

B: coordinating modifies illustration nonsense

NOTE: X, coordinating X is the subject (S) that does the coordinating job

X, coordinated X is the object (O) that was coordinated

A+B out

D: coordinated, BP BP is coordinated?? Shee is the one who coordinates, not the object out
E: E is the most tempting one (stupid Toan died at this). Read a bit further: illustrations is in the
prepositional modifiers them cannot take illustrations as an antecedent. (nu them i din cho
illustrations th cu nghe s rt k cc: BP coordinated the illustrations (with X) in her illustrations??

NOTE: dont use the noun in a prepositional modifier as an antecedent for them/they/it
107. While it costs about the same to run nuclear plants as other types of power plants, it is the
xed costs that stem from building nuclear plants that makes it more expensive for them to
generate electricity.

(A) While it costs about the same to run nuclear plants as other types of power plants, it is the
xed costs that stem from building nuclear plants that makes it more expensive for them to
generate electricity.

(B) While the cost of running nuclear plants is about the same as for other types of power plants,
the xed costs that stem from building nuclear plants make the electricity they generate more
expensive.

(C) Even though it costs about the same to run nuclear plants as for other types of power plants, it
is the xed costs that stem from building nuclear plants that makes the electricity they generate
more expensive.

(D) It costs about the same to run nuclear plants as for other types of power plants, whereas the
electricity they generate is more expensive, stemming from the xed costs of building nuclear
plants.

(E) The cost of running nuclear plants is about the same as other types of power plants, but the
electricity they generate is made more expensive because of the xed costs stemming from building
nuclear plants.

A+E: comparing the costs to run nuclear plants vs. other plants illogical

C+D: to run as for.. ungrammatical

A+C: makes does not agree with the notional noun preceding it: plants

D: stemming from is dangling

B is correct

109. Among the objects found in the excavated temple were small terra-cotta efgies left by
supplicants who were either asking the goddess Bona Deas aid in healing physical and mental ills or
thanking her for such help.

(A) in healing physical and mental ills or thanking her for such help

(E) to heal physical and mental ills or thanking her for such help

B+C+D: out

A vs. E: nu aid (n) c dng vi ngha l help with performing some activities th s cn in

Cn nu trong trng hp khc, khng ni r c th aid l help v ci g, ch l support normal

Eg: the midwife provided aid in childbirth;


The United States provides aid to many developing nations
111. A mutual fund having billions of dollars in assets will typically invest that money in hundreds of
companies, rarely holding more than one percent of the shares of any particular corporation.

(A) companies, rarely holding more than one percent

(B) companies, and it is rare to hold at least one percent or more

(C) companies and rarely do they hold more than one percent

(D) companies, so that they rarely hold more than one percent

(E) companies; rarely do they hold one percent or more

Stupid, Toan. It they are ambiguous

113. As a baby emerges from the darkness of the womb with a rudimentary sense of vision, it
would be rated about 20/500, or legally blind if it were an adult with such vision.

(A) As a baby emerges from the darkness of the womb with a rudimentary sense of vision, it would
be rated about 20/500, or legally blind if it were an adult with such vision.

(B) A baby emerges from the darkness of the womb with a rudimentary sense of vision that would
be rated about 20/500, or legally blind as an adult.

(C) As a baby emerges from the darkness of the womb, its rudimentary sense of vision would be
rated about 20/500; qualifying it to be legally blind if an adult.

(D) A baby emerges from the darkness of the womb with a rudimentary sense of vision that would
be rated about 20/500; an adult with such vision would be deemed legally blind.

(E) As a baby emerges from the darkness of the womb, its rudimentary sense of vision, which
would deemed legally blind for an adult, would be rated about 20/500.

A: it is ambiguous

B: latter part is awkward and distorts the meaning

C: fagment for the (;)

E: which distorts the meaning since it modifies vision

Only D left.
114. Starsh, with anywhere from ve to eight arms, have a strong regenerative ability, and if one
arm is lost it quickly replaces it, sometimes by the animal overcompensating and growing an extra
one or two.

(A) one arm is lost it quickly replaces it, sometimes by the animal overcompensating and

(B) one arm is lost it is quickly replaced, with the animal sometimes overcompensating and

(C) they lose one arm they quickly replace it, sometimes by the animal overcompensating,

(D) they lose one arm they are quickly replaced, with the animal sometimes overcompensating,

(E) they lose one arm it is quickly replaced, sometimes with the animal overcompensating,

A+C: agreement? out

If X, (then) Y X and Y must be // passive passive or active active D+E out

C + E: distort the meaning, and from this point, funny part starts.

C: they replace the lost arm, (sometimes by the animal overcompensating),growing extra one or
two. sometimes by the animal overcompensating = sometimes they grow extra arms but not by
the animal overcompensating? It always by the animal overcompensating.

E: they replace the lost arm, (sometimes w the animal overcompensating),growing extra one or
two So.. sometimes they grow extra arms without the animal overcompensating? Not the
intended meaning.

Also in C & E: they refers to starfish a species. If all of them lose 1 arm??? nonsense.
115. Because there are provisions of the new maritime code that provide that even tiny islets can
be the basis for claims to the sheries and oil elds of large sea areas, they have already stimulated
international disputes over uninhabited islands.

(A) Because there are provisions of the new maritime code that provide that even tiny islets can be
the basis for claims to the sheries and oil elds of large sea areas, they have already stimulated

(B) Because the new maritime code provides that even tiny islets can be the basis for claims to the
sheries and oil elds of large sea areas, it has already stimulated

(C) Even tiny islets can be the basis for claims to the sheries and oil elds of large sea areas under
provisions of the new maritime code, already stimulating

(D) Because even tiny islets can be the basis for claims to the sheries and oil elds of large sea
areas under provisions of the new maritime code, this has already stimulated

(E) Because even tiny islets can be the basis for claims to the sheries and oil elds of large sea
areas under provisions of the new maritime code, which is already stimulating

A: they refers to provisions? Areas? Fisheries? Provisions provide.. - redundant

C: what already stimulating disputes? D: this refers to what? This can never be a standalone
pronoun. E: fragment: only because clause is present.

RON:

COMMA + -ING does modify actions. that's what i mean by stating above that COMMA -ING
modifies the action of the preceding clause.
however, the fact remains that the -ING form is still derived from a verb, and so still needs to have
some sort of subject. in other words, the subject of the clause that's being modified must also
be the subject of the -ING form.
hence, why sentences like this are wrong:
using a siphon, gasoline can be transferred from a car's tank to a gas can.
--> here, "using a siphon" does modify the action of the following clause, but it doesn't have a
legitimate subject -- i.e., the following clause doesn't saywho is using a siphon.
vs. using a siphon, a stranded driver can transfer gasoline from a car's tank to a gas can.
--> here, the modifier correctly modifies the following action, but it also has a proper subject ("a
stranded driver").

--> when you use a COMMA -ING modifier after a clause**, you should actually satisfy TWO
requirements:
(1) the modifier should modify the action of the preceding clause, as you have stated shows
result or gives additional information;
(2) the subject of the preceding clause should also make sense as the agent of the -ING action.
examples:
Joe broke the window, angering his father. --> this sentence makes sense, because it correctly
implies that joe "angered his father".
the window was broken by Joe, angering his father. --> this sentence doesn't make sense,
because it implies that the window (i.e., not joe himself) angered joe's father.
my brother tricked me, disappointing Dad --> implies that dad is disappointed in my brother for
tricking me (and not necessarily disappointed in me for being tricked).
i was tricked by my brother, disappointing Dad --> implies that dad is disappointed in me because i
fell for my brother's trick (and not that he's disappointed in my brother for tricking me)

V-ing without a preceding coma modifies the directly preceding noun


117. Galls hypothesis of there being different mental functions localized in different parts of the
brain is widely accepted today.

(A) of there being different mental functions localized in different parts of the brain is widely
accepted today

(B) of different mental functions that are localized in different parts of the brain is widely accepted
today

(C) that different mental functions are localized in different parts of the brain is widely accepted
today

(D) which is that there are different mental functions localized in different parts of the brain is
widely accepted today

(E) which is widely accepted today is that there are different mental functions localized in different
parts of the brain

D+E: no comma before which out.

A: of there being is wordy

B: distorts the meaning implies there are other functions that are not localized

D: that there are.. wordy; E: implies that Gall has other hypos that are not accepted?

120. The World Wildlife Fund has declared that global warming, a phenomenon most scientists
agree to be caused by human beings in burning fossil fuels, will create havoc among migratory birds
by altering the environment in ways harmful to their habitats.

(A) a phenomenon most scientists agree to be caused by human beings in burning fossil fuels,

(B) a phenomenon most scientists agree that is caused by fossil fuels burned by human beings,

(C) a phenomenon that most scientists agree is caused by human beings burning of fossil fuels,

(D) which most scientists agree on as a phenomenon caused by human beings who burn fossil
fuels,

(E) which most scientists agree to be a phenomenon caused by fossil fuels burned by human
beings,

A: burning fossil fuels modifies human beings implying that these human beings are coming to a
most unfortunate end

B+E: the sentence says that the global warming results from the burning of the fossil fuels, not the
fossils themselves. B+E out

A+D: the sentence says that the global warming results from the burning of the fossil fuels (human
beings action), not from people themselves.
126. The use of lie detectors is based on the assumption that lying produces emotional reactions in
an individual that, in turn, create unconscious physiological responses.

(A) that, in turn, create unconscious physiological responses

(B) that creates unconscious physiological responses in turn

(C) creating, in turn, unconscious physiological responses

(D) to create, in turn, physiological responses that are unconscious

(E) who creates unconscious physiological responses in turn

Meaning: the sentence says that lying emotional reactions create responses. Common sense

B+C+D+E: imply that individual creates the responses

NOTE: for modifiers starting with that, the modified noun is not necessary to be placed
right before that. There might be a lot of stuff caught in between.

127. Joan of Arc, a young Frenchwoman who claimed to be divinely inspired, turned the tide of
English victories in her country by liberating the city of Orlans and she persuaded Charles VII of
France to claim his throne.

(A) she persuaded Charles VII of France to claim his throne

(B) persuaded Charles VII of France in claiming his throne

(C) persuading that the throne be claimed by Charles VII of France

(D) persuaded Charles VII of France to claim his throne

(E) persuading that Charles VII of France should claim the throne

A: The clause after "and" is an independent clause and thus requires a "," before the and. Or else
not //: she turned and she persuaded

B: Unidiomatic. "Persuaded X to do Y" is the correct idiomatic construction.

C: Unidiomatic. "Persuaded X to do Y" is the correct idiomatic construction. Also, you can only
persuade another person, not something (unanimated things such as throne)

D: Correct usage of parallel structure and idioms.

E: Same error as B and C. Also "should" implies moral obligation.


128. Australian embryologists have found evidence that suggests that the elephant is descended
from an aquatic animal, and its trunk originally evolving as a kind of snorkel.

(A) that suggests that the elephant is descended from an aquatic animal, and its trunk originally
evolving

(B) that has suggested the elephant descended from an aquatic animal, its trunk originally evolving

(C) suggesting that the elephant had descended from an aquatic animal with its trunk originally
evolved

(D) to suggest that the elephant had descended from an aquatic animal and its trunk originally
evolved

(E) to suggest that the elephant is descended from an aquatic animal and that its trunk originally
evolved

A: wordy and fragment

B: has wrong;

C+D: had wrong

130. Over 75 percent of the energy produced in France derives from nuclear power, while in
Germany it is just over 33 percent.

(A) while in Germany it is just over 33 percent

(B) compared to Germany, which uses just over 33 percent

(C) whereas nuclear power accounts for just over 33 percent of the energy produced in Germany

(D) whereas just over 33 percent of the energy comes from nuclear power in Germany

(E) compared with the energy from nuclear power in Germany, where it is just over 33 percent

A: it is an orphan.

B: compared to Germany is wrong, because Germany is not // to anything. France does not stand
alone. It is a part of a prepositional phrase in France cannot separate. Neither does France use
anything. 33% is ambiguous.

D: in Germany was misplaced distort the meaning.

E: it?? also, the comparison is ambiguous.


133. Last week local shrimpers held a news conference to take some credit for the resurgence of
the rare Kemps ridley turtle, saying that their compliance with laws requiring that turtle-excluder
devices be on shrimp nets protect adult sea turtles.

(A) requiring that turtle-excluder devices be on shrimp nets protect

(B) requiring turtle-excluder devices on shrimp nets is protecting

(C) that require turtle-excluder devices on shrimp nets protect

(D) to require turtle-excluder devices on shrimp nets are protecting

(E) to require turtle-excluder devices on shrimp nets is protecting

A+C+D: their compliance protect/are protecting: wrong!

E: to require is ambiguous.

Gprep 1.

Which of the following most logically completes the argument?


When people engage in activities that help others ,their brain releases endorphins, the brains
natural opiates, which induce in people a feeling of well-being. It has been suggested that regular
release of endorphins increases peoples longevity. And a statistic on adults who regularly engage in
volunteer work helping others shows that they live longer, on average, than adults who do not
volunteer. However, that statistic would be what we would expect even if volunteering does not
boost longevity, because __________.
A. in the communities studied, women were much more likely to do regular volunteer work than
men were, and women tend to live longer than men do.
B. the number of young adults who do regular volunteer work is on the increase
C. the feelings of well-being induced by endorphins can, at least for a time, mask the symptoms of
various conditions and diseases, provided the symptoms are mild.
D. it is rare for a person to keep up a regular schedule of volunteer work throughout his or her life.
E. Some people find that keeping a commitment to do regular volunteer work becomes a source of
stress in their lives.

Cu ny b vng fallacy l correlation: t vic thng k ngi tham gia tnh nguyn c tui th cao
hn a ra kt lun v mi quan h nhn qu: X & Y xy ra ng thi kt lun X gy ra Y. C
2 yu t lm yu nhn nh ny:

(1). Y gy ra X

(2). C 2 u c chung 1 nguyn nhn th 3

yu t gii tnh a ra trong cu A gii thch (2) correct!


The investigations of many psychologist and anthropologists support the generalization of there
being little that is a significant difference in underlying mental processes manifested by people from
different culture.
A. of there being little that is a significant difference
B. of there being little that is significantly different
C. of little that is significantly different
D. that there is little that is significantly different
E. that there is little of significant differences

The generalization THAT is idiomatic, The generalization of is unidiomatic A+B+C out

Also, there being is at best awkward A+B+C out

Little in D & E is a noun, D is ok

E: little and differences do not match!

Trans World Entertainment Corporation, which owns the RecordTown and Saturday Matinee retail
chains, announced that since sales of up to one-fourth of its stores are poor, they will be closed
A. that since sales of up to one-fourth of its stores are poor, they will be closed
B. it is closing up to one-fourth of its stores, which accounted for its poor sales
C. it was closing up to one fourth of its stores because of poor sales
D. to be closing, on account of poor sales, up to one-fourth of its stores
E. having poor sales, such that up to one-fourth of its stores will be closed

C is correct

A: they is ambiguous what will be closed? All of the store or of the stores?

+ weird meaning: sales of X X is sold. In this case, stores??

B: verb tense does not make sense + which is ambiguous + changes the meaning

D: wordy; to be doing something is at best weird

E: having poor sales is a disaster, and right after the (,) was not the noun.

Since the start of space age, more and more littering has occured in orbits near Earth, often
because the intentional discarding of lens caps, packing material , fuel tanks and payload covers.
a) more and more littering has occured in orbits near Earth, often because
b) orbits near Earth have become more and more littered, often from
c) orbits near Earth became littered more and more, often resulting from
d) there have been more and more littering of orbits near Earth, often because of
e) there had been littering more and more of orbits near Earth, often with

Key thing, Ton: orbit = qu o, path, khng phi v tinh T_T

A: because modifies a clause and is followed by a clause out! More and more littering is awkward
C: since starts in the past + the effect still shines through present present perfect C+E out

Also: resulting takes no noun to be an agent (orbits??)

D: littering cant take the plural form of have out

In contrast to ongoing trade imbalances with China and Japan, the United States trade deficit with
Mexico declined by $500 million as a result of record exports to that country.

A. In contrast to ongoing trade imbalances with China and Japan, the United States trade deficit
with Mexico declined by $500 million as a result of record exports to that country.
B. In contrast to ongoing trade imbalances with China and Japan, the United States sold record
exports to Mexico, reducing its trade deficit by $500 million.
C. When compared with ongoing trade imbalances with China and Japan, the United States sold
record exports to Mexico, reducing their trade deficit by $500 million.
D. Compared with ongoing trade imbalances with China and Japan, the United States sold record
exports to Mexico, reducing the trade deficit by $500 million.
E. Compared to ongoing trade imbalances with China and Japan, the United States record exports to
Mexico caused a $500 million decline in the trade deficit with that country.

B+C+D: comparing trade imbalances with a country out!

E: comparing trade imbalances with exports nonsense out!

Dirt roads may evoke the bucolic simplicity of another century, but financially strained townships
point out that dirt roads cost twice as much as maintaining paved roads. (comparison problem)

(A) dirt roads cost twice as much as maintaining paved roads

(B) dirt roads cost twice as much to maintain as paved roads do

(C) maintaining dirt roads costs twice as much as paved roads do

(D) maintaining dirt roads costs twice as much as it does for paved roads

(E) to maintain dirt roads costs twice as much as for paved roads

A: dirt roads vs. maintaining wrong

C: maintaining dirt roads vs. paved roads wrong

D: it has no antecedent; D+E: awkward comparison, should be: maintaining dirt roads costs twice
as much as maintaining paved roads does or to maintain X costs twice as to maintain Y
Only seven people this century have been killed by the great white shark, the man-eater of the
movies-less than those killed by bee stings. (comparison problem)

a. movies--less than those


b. movies--fewer than have been
c. movies, which is less than those
d. movies, a number lower than the people
e. movies, fewer than the ones

A + C out for less

D: compare number with people?

E: in GMAT, ones cannot refer to people

because the more X than Y construction is workable with just about any grammatical forms for "x"
and "y". They can be...
nouns (i have more apples than bananas)
verbs (more people swim than run)
infinitives (i like to swim better than to run)
verb phrases (like the sentence in this thread)
entire clauses (birds like to sing more than penguins like to dance)
... and just about anything else under the sun. so, who cares, really, as long as you can find properly
parallel structures.

Even an imaginary version: fewer than those killed by is wrong because we can never say: sth
is fewer than other things (applied to EVERYTHING): men are fewer than womenwrong!

There are fewer men than women in this city correct

This city has fewer men than women correct E is wrong also for this reason.

More on comparison:

Remember, the exact entities being compared are not always found on both sides of a comparison.
Instead, it's the parts that differ that will always be found on both sides"because they're different.

In New York, more people walk than drive to work.


We're comparing numbers of people. The parts that are actually different, though, are "walk" and
"drive", so those are the things that actually appear in the comparison.

I saw more birds in the Philippines than in Malaysia.


We're comparing the numbers of birds that I saw. The parts that differ, though, are "in the
Philippines" and "in Malaysia""so those are written out.

Some examples:
139. In no other historical sighting did Halleys Comet cause such a worldwide sensation as did its
return in 1910 1911. (comparison problem)

(A) did its return in 19101911

(B) had its 1910 1911 return

(C) in its return of 19101911

(D) its return of 19101911 did

(E) its return in 19101911

Note: if you have 2 ANSWER CHOICES THAT DIFFER ONLY IN THE PRESENCE/ABSENCE OF A
LITTLE WORD, then CHANCES ARE THAT THE LITTLE WORD IS NECESSARY.
this is a really, really, really strong bias, like over 90% likely. if you see two such choices, then you
should REALLY think twice before picking the one without the "little word"
i can't guarantee this 100%, but LOOK FOR AN AMBIGUITY that arises if the little word is omitted.

Back to this question, the main point is to figure out the things that are being compared. In the not-
underlined part, there are 2 parts:

* the comet DID cause a sensation


* IN no other sighting
so if you had "[some other sighting] DID", or "IN [something else that caused a sensation]", then that
would be an error -- not necessarily a grammar error, but a meaning error because it'd be nonsense.
on the other hand, if you saw "IN [some other sighting]", or [something else that caused a sensation]
DID", then that would create a valid comparison.
this, by the way, is the reason why choices (a) and (d) are wrong. they aren't grammatical errors;
they just set the wrong things up to be compared.

In no other sighting did X cause such a sensation as it did in its return of 1910-1911 correct

In no other sighting did X cause such a sensation as in its return of 1910-1911 correct

The latter one is not necessarily a partern of ellipsis, it compares 2 events: no sighting vs the return

Other examples:

I am better at SC now than 2 months ago = I am better at SC now than I was 2 months ago

(the 1st part is not an ellipsis since we can only omit the exact matching words, no tense change)

In no game this season has Harold made such a spectacular play as in last year's championship
game --> correct; the intended comparison is between the two games.

In no game this season has Harold made such a spectacular play as last year's championship
game --> incorrect; the intended comparison is between the two games, but the sentence mistakenly
classifies last year's championship game as "a spectacular play".
Read this for more info: https://www.manhattanprep.com/gmat/forums/ron-s-advice-on-comparison-
and-omission-t31036.html
COMPARISON AND OMISSION - RON

for sentences that don't have such a simple structure, the goal is to avoid introducing unnecessary
complexity.

here's what you should do with such sentences:


1/ identify the right-hand element (= the one that follows the comparison signal)
2/ ask yourself whether this element SHOULD be part of the comparison, according to
context/common sense. (there are no "omissions" here. if the stuff following the signal isn't actually
compared to something else, the sentence is wrong.) (TOAN: so omission is only used when the
sentence is too complicated?)
3/ if so, try to find something that matches it on the other side. if you find such a thing, you're good.

some examples related to your first 2 sentences:


People are taller in China than Spain.
1/ the right-hand element is "spain".
2/the sentence is not intended to compare spain itself with anything. so, this version is wrong. quit.
--
People are taller in China than in Spain.
1/ the right-hand element is "in spain".
2/ the sentence compares heights of people, sure. however, it's also true that the sentence directly
compares the situation in china with the situation in spain. (it does not, on the other hand, compare
the actual countries of china and spain; hence why the foregoing example is wrong.)
so, this works.
3/ "in china" is there. this one works.
--
gas prices were higher in 2007 than in 1997.
1/ right-hand element = "in 1997"
2/ this is legitimate (see write-up of previous example, which works in exactly the same way)
3/ "in 2007" is there; we're good.
--
gas prices were higher in 2007 than those in 1997.
1/ right-hand element = "[gas prices] in 1997" (meaning of the pronoun is obvious, i hope)
2/ we want to compare prices, so the set-up is ok
3/ nothing works on the other side. we need "gas prices in 2007" (in one piece), but only "gas prices"
is there. nope.
--
gas prices in 2007 were higher than those in 1997.
this is one of the easy ones: "X were higher than Y".
X = gas prices in 2007
Y = [gas prices] in 1997 workable.
Caribou are wary animals with excellent hearing, so stalking them over the treeless landscape,
getting close enough to kill it with nothing but a handheld lance, as Dorset people did, required
exceptional hunting skill.

(A) so stalking them over the treeless landscape, getting close enough to kill it

(B) so to stalk them over the treeless landscape and get close enough to kill one

(C) so in order to stalk them over the treeless landscape and get close enough to kill one

(D) and so in order to stalk it over the treeless landscape, getting close enough to kill it

(E) and so stalking them over the treeless landscape and getting close enough in order to kill it

A+D+E: out for it. Caribou are

B: to V can stand as a subject, but in order to V cannot

2. According to a survey of graduating medical students conducted by the Association of American


Medical Colleges, minority graduates are nearly four times more likely than are other graduates in
planning to practice in socioeconomically deprived areas.
(A) minority graduates are nearly four times more likely than are other graduates in planning to
practice
(B) minority graduates are nearly four times more likely than other graduates who plan on practicing
(C) minority graduates are nearly four times as likely as other graduates to plan on practicing
(D) it is nearly four times more likely that minority graduates rather than other graduates will plan to
practice
(E) it is nearly four times as likely for minority graduates than other graduates to plan to practice

A: LIKELY IN PLANNING WRONG

B: more likely to do what? Also, this sentence only compares 2 numbers

D: more but lack of than. More likely than slashes with rather than

E: as than?
With no natural predators and expanses of green suburban neighborhoods that allow no hunting,
wildlife officials estimate the New Jersey deer population to have grown to exceed 175,000.

A. With no natural predators and expanses of green suburban neighborhoods that allow no hunting,
wildlife officials estimate the New Jersey deer population to have

B. With no natural predators and with expanses of green suburban neighborhoods that do not allow
hunting, wildlife officials' estimate of deer population in New Jersey has

C. With no natural predators and with expanses of green suburban neighborhoods where there is no
hunting, the Deer Population in New Jersey, wildlife officials estimate, has

D. Without natural predators and no hunting allowed in expanses of green suburban neighborhoods,
New Jersey has a deer population that wildlife officials estimate to have

E. Without natural predators and with expanses of green suburban neighbourhoods where there is
no hunting, wildlife officials in New Jersey estimate a deer population that has

Without/with no predators cannot modify wildlife officials or New Jersey. Obviously, there are no
predators threaten to eat New Jersey or the officials (or officials estimate).

Though using this opening modifier to modify deer population is still ridiculous, this usage is the
least ugly choice presented. You might understand the sentence as: without predators in New
Jersey but lets narrow it down:

Without predators, New Jersey has many deers.

Still sounds awkward to think that in NJ, not only the deers but also all other animals do not have
any predators distorts the meaning of the sentence.

In A + D, parallelism is broken out

Other eg with with modifiers

"I have nowhere to go" --> With nowhere to go, I can stay home and relax.
"There is no tree in which the bird can make a nest" --> With no tree in which to make a nest, the
bird must construct one atop some man-made structure.
GMAT PREP PACK 2

Introduced by Italian merchants resident in London during the sixteenth century, in England life
insurance remained until the end of the seventeenth century a specialized contract between
individual underwriters and their clients, typically being ship owners, overseas merchants, or
professional moneylenders.

A. in England life insurance remained until the end of the seventeenth century a specialized contract
between individual underwriters and their clients, typically being
B. in England life insurance had remained until the end of the seventeenth century a specialized
contract between individual underwriters with their clients, who typically were
C. until the end of the seventeenth century life insurance in England had remained a specialized
contract between individual underwriters and their clients, typically
D. life insurance in England remained until the end of the seventeenth century a specialized contract
between individual underwriters and their clients, typically
E. life insurance remained until the end of the seventeenth century in England a specialized contract
between individual underwriters with their clients, who typically were

Make sure you get the takeaway here: in the case of an initial subject-less modifier in front of a
comma, the modifier should always be followed IMMEDIATELY by the noun / noun phrase
that's being modified.
there should not be any intervening constructions such as the prepositional phrase that appears
here. totally unacceptable.

Therefore A + B +C: out

Also, until + past tense B+C out; A: being is reduntdant (can be omitted)

E: distort the meaning: life insurance remained in England; between + and, not with
espite the growing number of people who purchase plane tickets online, airline executives are
convinced that, just as one-third of bank customers still prefer human tellers to automatic teller
machine, many travelers will still use travel agents

A) growing number of people who purchase plane tickets online, airline executives are convinced
that, just as one-third of bank customers still prefer human tellers to automatic teller machines, many
travelers will

B) growing number of people who purchase plane tickets online, airline executives are convinced,
just as one-third of bank customers still prefer human tellers to automatic teller machines, that many
travelers would

C) growing number of people purchasing plane tickets online, airline executives are convinced, just
as one-third of bank customers still prefer human tellers as compared to automatic teller machines,
many travelers will

D) fact that the number of people purchasing plane tickets online is growing, airline execituves are
convinced, just as one-third of bank customers still prefer human tellers as compared to automatic
teller machines, that many travelers would

E) fact that the number of people who purchase plane tickets online are growing, airline executives
are convinced that, just as one-third of bank customers still prefer human tellers compared with
automatic teller machines, many travelers would

B+C+D: executives are convinced, just as X do Y comparing 2 actions distort the meaning.
THAT needs to be placed right after convinced, otherwise, it makes reader believe that both
executives and customers are convinced.

D+E: despite the fact that = wordy

E: number of people are = doesnt agree

C+D+E: prefer TO, not compared to/with

When drive-ins were at the height of their popularity in the late 1950s, some 4,000 existed in the
United States, but today there are less than one-quarter that many (comparison problem)
A. there are less than one-quarter that many
B. there are fewer than one-quarter as many
C. there are fewer than one-quarter of that amount
D. the number is less than one-quarter the amount
E. it is less than one-quarter of that amount

We are comparing countable noun (drive-ins) less is wrong A+D+E out.

Remember: we have to use the struture as as when we did not mention the data earlier in
the sentence: I have twice/ 3 times/ one-quarter/ half as many apples as I had last week

But if the information was mentioned earlier, only 1 as is needed (ellipsis is legal):
Last week I had 20 apples, now I have only one-quarter as many

I found 12 words on the puzzle game, but my brother found even more.

Amount is used for uncountable nouns C+D+E out.

18. Today's techonology allows manufacturers to make small cars more fuel efficient now than at
any time in their production history. (comparison problem)

a. small cars more fuel efficient now than at any time in their
b. small cars that are more fuel-efficient than they were at any time in their
c. small cars that are more fuel-efficient than those at any other time in
d. more fuel-efficient small cars than those at any other time in their
e. more fuel-efficient small cars now than at any time in

Seriously, this one again test you about how you understand the sentence: what kind of things are
being compared? Now v.s in the past? Cars?

the best: small cars now are more fuel efficient than the cars in the past.

If you compare now v.s the past U compare the exact same small cars: they are now more
efficient than THEY were years ago? NO! cars now and then are different cars!

19. Recently documented examples of neurogenesis, the production of new brain cells, include the
brain growing in mice when placed in a stimulating environment or neurons increasing in canaries
that learn new songs. (parallelism) - 700

A. the brain growing in mice when placed in a stimulating environment or neurons increasing in
canaries that
B. mice whose brains grow when they are placed in a stimulating environment or canaries whose
neurons increase when they
C. mice's brains that grow when they are placed in a stimulating environment or canaries' neurons
that increase when they
D. the brain growth in mice when placed in a stimulating environment or the increase in canaries'
neurons when they
E. brain growth in mice that are placed in a stimulating environment or an increase in neurons in
canaries that

RON: by the way, i really, really hate this question. i don't think "or" has any business being in this
sentence; it should clearly be "and", because both of these are recently documented examples of
neurogenesis. i don't understand how they can write "or" with a straight face. but, as we've said so
many times on here, it's their playground, and they make the rules.

A: breaks parallelism: X when placed in specific environment or Y that learn new songs

B+C: mice cannot be example of neurogenesis, should be mices brain growth.

B+C+D: ambiguous they

A+B+C+D: meaning mice brain only develop when put in a specific envi? Nonsense.
D: not parallel: when placed vs. when they learn
Lyme disease is caused by a bacterium transmitted to humans by deer ticks. Generally deer ticks
pick up the bacterium while in the larval stage from feeding on infected white-footed mice. However,
certain other species on which the larvae feed do not harbor the bacterium. Therefore, if the
population of these other species were increased, the number of ticks acquiring the bacterium would
likely decline.
Which of the following would be most useful to ascertain in evaluating the argument?

(A) Whether populations of the other species on which deer tick larvae feed are found only in the
areas also inhabited by white-footed mice.
(B) Whether the size of the deer tick population is currently limited by the availability of animals for
the tick's larval stage to feed on.
(C) Whether the infected deer tick population could be controlled by increasing the number of
animals that prey on white-footed mice.
(D) Whether deer ticks that were not infected as larvae can become infected as adults by feeding on
deer on which infected deer ticks have fed.
(E) Whether the other species on which deer tick larvae feed harbor any other bacteria that ticks
transmit to humans.

Line of reasoning: bacterium deer ticks human

Bacterium white-footed mice tick larvaes human

No bacterium Other species not infected larvaes

Increase other species ticks acquiring bacterium decline???????

In B: If (B) is a "yes", this means that the tick population is smaller than it could be, because there
aren't enough host animals to go around. I.e., the tick population "wants" to get bigger, but simply
can't, because there aren't enough host animals.
If that's the case, then introducing extra host animals won't spread the ticks out more thinly; it will just
increase the total number of ticks! So the bacterium won't be affected.

Negative:
If (B) is a "no", then the tick population is not artificially small as a result of a imited # of host
animals. In that case, the plan is likely to have the described effect. (the tick population is small
already more food source spread out ticks lower number of infected ticks. Ticks will not
likely to ignore new food source)

DONT PUT IN TOO MUCH THOUGHT!!!!!!!!!!!!


GMAT PREP PACK 2 (5)

In 1981 children in the United States spent an average of slightly less than two and a half hours a
week doing household chores; by 1997 they had spent nearly six hours a week. (comparison
problem)

A. chores; by 1997 they had spent nearly six hours a week

B. chores; by 1997 that figure had grown to nearly six hours a week

C. chores, whereas nearly six hours a week were spent in 1997

D. chores, compared with a figure of nearly six hours a week in 1997

E. chores, that figure growing to nearly six hours a week in 1997

A: They here means the children in 1981 nonsense. Also, it makes no sense to say that they
(children) had spent nearly 6h per week The intended meaning is:
In 1981 the children spent X hours a week for some work.
In 1997 the children spent Y hours a week for some work.

For option A, first exclude the part "per week" - option A then would imply that when the year 1997
came, children had already spent certain number of hours working. (compare with this sentence: By
3 pm I had already completed 3 hours of exercise - meaning: I had been exercising from 12 noon to
3 pm). This meaning itself is wrong (it is not meant that the children had already completed certain
number of hours by 1997). However addition of "per week" makes the sentence senseless
altogether. (compare with: By 3 pm I had already completed 3 hours of exercise per day.)

C: 6 hours/week to do what? + break parallelism

D: compared with: should be parallelism

E: you can't use pointing words like "that" and "this" in modifiers that immediately abut the
nouns/phrases that they modify, separated only by a comma. instead, you have to use a different
sort of noun modifier. E out for that
illustrations:
the coach ran a play with five receivers on the line, and this strategy succeeded nicely --> ok
(although a bit awkward in this case).
the coach ran a play with five receivers on the line, this/that strategy succeeding nicely --> incorrect
type of modifier for this situation.
the coach ran a play with five receivers on the line, a strategy that succeeded nicely --> correct type
of modifier for this situation.

KH VI CHNG RA!!!!!!
Most states impose limitations on the authority of the legislature to borrow money, with their
objectives being to protect taxpayers and the credit of the state government.
A. to borrow money, with their objectives being to protect
B. to borrow money, the objectives of which are the protecting of
C. to borrow money, limitations intended to protect
D. for borrowing money, of which the objective is protecting
E. for borrowing money, limitations with the intent of protecting

A: objectives plural? Being to is wordy

B: objectives.

D+E:limitations to is correct. Limitations for is incorrect.

E: limitations cannot have intent

New items developed for automobiles in the 1997 model year included a safer air bag, which, unlike
previous air bags, eliminated the possibility that a burst of smoke would appear when the
bag inflated, and making an already terrified passenger think the car was on fire.
A. inflated, and making
B. inflated, so that it could make
C. inflated and made
D. inflated and make
E. inflated to make

A: and makes this one wrong. No parallel structure

B + E: intended? Nonsense.

C: the bag inflated and made passenger think the car was on fire? No, what make a passenger think
of a fire is the burst of smoke. make (D). parallel: the smoke would appear and make

In two months, the legal minimum wage in the country of Kirlandia will increase from five Kirlandic
dollars(KD5.00) Per hour to KD5.50 per hour. Opponents of this increase have argued that the
resulting rise in wages will drive the inflation rate up. In fact its impact on wages will probably be
negligible, since only a very small proportion of all Kirfandic workers are currently receiving less than
KD5.50 per hour.
Which of the following, if true, most seriously weakens the argument?
A. Most people in kirlandia who are currently earning the minimum wage have been employed at
their current jobs for less than a year. irrelevant
B. Some firms in Kirlandia have paid workers considerably less than KD5.00 per hour, in violation of
kirlandic employment regulations.
C. Many businesses hire trainees at or near the minimum wage but must reward trained workers by
keeping their paylevels above the pay level of trainees.
D. The greatest growth in Kirlandias economy in recent years has been in those sectors where
workers earn wages that tend to be much higher than the minimum wage.
E The current minimum wage is insufficient for a worker holding only one job to earn enough to
support a family ,even when working full time at that job.

C: higher minimum wage affect not only the small proportion of workers who are now receiving
minimum wage, but also other people in companies (since they are trained workers)
Although the cacao plant probably originated in the upper Amazon region, its first cultivation for seeds
and pulp was apparently begun in Mexico and Central America.

A. its first cultivation for seeds and pulp was apparently begun
B. apparently it was cultivated for its seeds and pulp beginning
C. it was apparently first cultivated for its seeds and pulp
D. the beginning of its cultivation for seeds and pulp appears to be
E. the first cultivation for its seeds and pulp appears to have begun

A: the cultivation was begun in Mexico and then was concluded somewhere else? nonsense

B: beginning modifies pulp??wq

D: the beginning appears - present tense? Its in the past, Toan

E: same with begun and appea

Like the Dreyfus affair at the century and the Sacco-Vanzetti trial in 1921, the Hiss case became the
political cause of an era, the more so because it was the first time a congressional hearing was shown on
television. (comparison problem)

A. the more so because it was


B. more for its being
C. more for the reason that it was
D. the more so for the reason because it was
E. the reason of it being more so is that it was

Here, usage of the more so implies the following comparison:


Because the congressional hearing was shown on television, the Hiss case became MORE of a political
cause THAN it would have become (had the hearing NOT been shown on television).
Even though the comparison is not explicitly stated, the usage of the more so makes the implied
comparison crystal clear.

This usage may be considerd as stating an action and then providing an additional reason for taking the
action - a reason that made someone take the action with stronger justification.

I went to the party, more so because she was there.


GMAT PREP PACK 2 (6)

Though the law will require emissions testing of all diesel vehicles, from tractor trailers to excursion
buses, it will have no effect on sport utility vehicles, almost all of which are gasoline powered, and
will not be sub-jected to emissions-control standards as stringent as they are for diesel-powered
vehicles. (comparison problem)
A. powered, and will not be subjected to emissions-control standards as stringent as they are for
B. powered, and therefore not subjected to emissions-control standards that are as stringent as
those of
C. powered and therefore not subject to emissions-control standards as stringent as those for
D. powered, which are not subject to emissions-control standards as stringent as they are for
E. powered and therefore they are not subject to emissions-control standards as stringent as those
of

Again, this problem tests you on how you understand the meaning.

OF vs. FOR are totally different! X of Y Y itself has X; X for Y X is externally imposed on Y!!!

B+E out.

Be Subjected to X: be forced to undergo some rules/regulations

Be subject to X: rules that apply to X

A+B out

D: which is wrong

Cn hiu cu ny nh sau: mc d lut quy nh cn c cc bi kim tra mc x thi nhng phng


tin chy du diesel, lut ny s khng c tc dng vi nhng phng tin sd trong th thao v chng
chy xng v v vy khng phi tun theo nhng qui nh cht ch nh quy nh dnh cho phng tin
chy du. FUCK! chng n ang so snh standards ca 2 thng, coi thng no nghim ngt hn thng
no, ch khng so snh 2 loi xe.

Financial analyst said they were surprised that despite higher inflation and shrinking profits margins,
investors continue pouring money into the stock market, believing that the American economy
was nicely balanced, with little inflation.

(a) continue pouring money into the stock market, believing that the American economy was

(b) continue pouring money into the stock market, believing in the American economy as

(c) still continue pouring money into the stock market in the belief that the American economy as

(d) still continued pouring money into the stock market, believing in the American economy as

(e) continued to pour money into the stock market in the belief that the american economy was

The analysts said in the past continued shoul be used, A+B+C out
D: still is redundant. Believing in something is ok, but believing in X as Y is incorrect. E!
With California expected to see severe electricity shortfalls and perhaps blackouts on as many as 30
days this summer, the administration has grown increasingly concerned about public health and
safety there. (Comparison problem)

A. With California expected to see severe electricity shortfalls and perhaps blackouts on as many as
30 days this summer, the administration has grown increasingly concerned about

B. With California expecting to see severe electricity shortfalls this summer, and there will possibly
be blackouts for as much as 30 days, the administration's concern has grown increasingly about

C. As California is expected to be seeing severe electricity shortfalls and perhaps blackouts on as


much as 30 days this summer, the administration's concern is increasing for

D. Insofar as California is expected to see severe electricity shortfalls this summer, and there will
possibly be blackouts on as many as 30 days, the administration has increasing concern about

E. Insofar as California expects to see severe electricity shortfalls and the possibility of blackouts for
as much as 30 days this summer, the administration has grown increasingly concerned about

Days countable B+C+E out for much

D removes summer after 30 days distort the meaning; insofar = to the extent that..., but
insofar as does not make much sense.

Bumblebees live in colonies of several hundred that often have many queens as well as a large
number of workers

A) of several hundred that often have many queens as well as


B) of several hundred, often with many of them being queens as well as having
C) having several hundred, and often many of them are queens as well as
D) that have several hundred of them, often with many queens as well as

E) with several hundred that often have many queens as well as having

Parallel B+E out

C: ambiguity

D: distorts the meaning: bumblebees only live in colonies THAT have several hundred of them? They
dont live in other colonies that have more or fewer bees? Also, D makes us think that bumblebees
live with queens and workers they are not queens or workers?? nonsense.
GMAT PREP PACK 1 (3)

Researchers hypothesize that granitic soil is the ideal construction material for the desert tortoise
because it is not so hard that it makes burrowing difficult or so soft that it could cause tunnels to
collapse.

A. so hard that it makes burrowing difficult or so soft that it could cause


B. hard enough to make burrowing difficult or soft enough as to cause
C. so hard as to make burrowing difficult or soft enough so it causes
D. as hard as to make burrowing difficult or as soft as to cause
E. too hard, making burrowing difficult, nor too soft, so as to cause

B: as to.. unidiomatic

C: soft enough so: unidiomatic; so hard as tosoft enough so. : not //

D: as hard as to do: unidiomatic as as needs 2 nounds to compare

E: not // only A

Citing the recent increase in earnings by several computer companies, economists feel that a cycle
has begun in which personal computer users, especially corporate consumers, are replacing their
PCs with more powerful models.
A. a cycle has begun in which personal computer users
B. a cycle for personal computer users has begun in which they
C. there is a cycle beginning for personal computer users
D. it is the beginning of a cycle for personal computer users
E. personal computer users are in the beginning of a cycle when they

C+D: for users are replacing? out

B: distorts the meaning

E: distorts the meaning

This particular sentence deals with a cycle in which PC users are doing xxxx.
"Has begun" is the main verb; stripped of all modifiers, the sentence reads as "A cycle has begun."

Because the modifier (in which PC users are doing xxx) is sooooooo long, the usual order of the
verb and modifier (e.g., A cycle of new purchases has begun) is reversed.

You'll never be tested on this kind of ordering issue; since it's an issue of style and easy readability,
not of actual right and wrong, it lies beyond the scope of this exam.
However, you need to know at least that it's permissible at timesin other words, that it's a thing that
exists in written English. That way you won't erroneously mark it as wrong when you see it.
WORDS that change numbers (Egmat):

A. Each

EACH: nu trong cu c each verb chia ty theo v tr ca each:

Each pronoun: always singular:

Each of the 4 students is supposed to be on time (of the students: preposition phrase)

Each + plural nouns = singular

Eg: EACH condiment and herb brings a distintive flavor to the dish

Ging every: every boy and girl has to join the meeting

Each after the subject: depends on the subject

The four students each are supposed to be on time

B. Either/neither:

nu khng i cng or/nor: lun singular:

Eg: either shirt is fine;

Eg: neither of the two men is gay

Nu i cng or/nor: chia theo dt gn nht

Eg: either the students or the teachers hate the shared toilet

C. Additive phrases

Additive phrases like Along with, accompanied by, in addition to, as well as, toghether with do not
change the number of subject, no matter where they appear in the sentences:

Eg: along with the students, the teacher hates the toilet main S: teacher singular

Eg: Jim, accompanied by his friends, finish the route in time.

Discussion of greenhouse effects have usually had as a focus the possibility of Earth growing
warmer and to what extent it might, but climatologists have indicated all along that
precipitation, storminess, and temperature extremes are likely to have the greatest impact on
people.

(A) Discussion of greenhouse effects have usually had as a focus the possibility of Earth growing
warmer and to what extent it might,
(B) Discussion of greenhouse effects has usually had as its focus whether Earth would get warmer
and what the extent would be,

(C) Discussion of greenhouse effects has usually focused on whether Earth would grow warmer and
to what extent,

(D) The discussion of greenhouse effects have usually focused on the possibility of Earth getting
warmer and to what extent it might,

(E) The discussion of greenhouse effects has usually focused on whether Earth would grow warmer
and the extent that is,

A+D: have out

B: wrong idiom: The current construction requires a noun to follow the word focus.
Michael has usually maintained personal growth as his focus. (Correct)
Michael has usually maintained as his focus personal growth. (Rephrase: Correct)
Michael has usually maintained as his focus how to improve his personal growth.
(Clause: Incorrect)
Similarly, in this sentence, the usage of the clause whether Earth would get after focus is
grammatically incorrect.
Also, remember that wh- clause should be put in // with another wh- clause. The Wh words
are more like one another than they are to other words. E: not parallel since whether earth is
not // with the extent

While many of the dinosaur fossils found recently in northeast China seem to provide evidence of
the kinship between dinosaurs and birds, the wealth of enigmatic fossils seem more likely at this
stage that they will inflame debates over the origin of birds rather than settle them.

(A) seem more likely at this stage that they will inflame debates over the origin of birds rather than
(B) seem more likely that it will inflame debates over the origin of birds at this stage than
(C) seems more likely to inflame debates on the origin of birds at this stage rather than
(D) seems more likely at this stage to inflame debates over the origin of birds than to
(E) seems more likely that it will inflame debates on the origin of birds at this stage than to

A+B: the wealth... seem out

B+C+E: the placement of at this stage is wrong, leading to change of the meaning

E: will inflame does not // with settle them


The gyrfalcon, an Arctic bird of prey, has survived a close brush with extinction; its numbers are
now five times greater than when the use of DDT was sharply restricted in the early 1970s.
(comparison problem)

(A) extinction; its numbers are now five times greater than

(B) Extinction; its numbers are now five times more than

(C) Extinction, their numbers now fivefold what they were

(D) Extinction, now with fivefold the numbers they had

(E) Extinction, now with numbers five times greater than

(cu ny anh chn p n C v hnh nh v sau cu ny cng c add thm vo b prep,


nhng cu C c iu chnh khc hn so vi bn test code 42)

B: number + greater, not more

C+D: wrong pronoun

E: if you're going to use "now" + some phrase as a modifier, then "now" must accurately modify the
action and timeframe of the preceding clause.
e.g.
James is expanding the scope of his job search, now looking in Malaysia and Thailand as well as in
Singapore
--> this sentence is correct, because "now looking in..." actually modifies (describes) the way in
which james "is expanding his search".
here, now with numbers five times greater than does NOT modify the idea of "surviving a close
brush with extinction"; instead, it is a much later situation, far removed from (and in considerable
contrast to) the near-extinction. therefore, a modifier is inappropriate here; a separate clause, written
in a different tense (as in the correct answer), is a better solution.

(tc l: the 2nd part of the sentence does not act as modifier, but a contrast with the previous clause)

The European Economic Community exported just under $400 billion in goods in 1988, sixty percent
more than the United states did and nearly twice as much as Japan's exports (comparison problem)

A. the United states did and nearly twice as much as Japan's exports
B. the United States' and nearly double what the Japanese exports were
C. the United States exported and nearly twice as much as the Japanese did
D. what the United States did and nearly twice as much as Japan's exports.
E. what the United States exported and nearly double the Japanese exports

Only in C can we find parallel structure


Unlike the low status accorded the craftwork of many women settlers in the nineteenth-century
American West, Native American women of the Plains tribes often received public recognition,
wealth, and prestige for their craftwork. (comparison problem)

(A) Unlike the low status accorded the craftwork of many women settlers in the nineteenth-century
American West, Native American women of the Plains tribes
(B) Unlike many women settlers in the nineteenth-century American West, whose craftwork was
accorded low status, Native American women of the Plains tribes
(C) Native American women of the Plains tribes, unlike the craftwork of many women settlers in the
nineteenth-century American West,
(D) The craftwork of many women settlers in the nineteenth-century American West was accorded
low status, while that of the Native American women of the Plains tribes
(E) While low status was accorded the craftwork of many women settlers in the nineteenth-century
American West, that of Native American women of the Plains tribes

A: low status is being compared to women

C: women is being compared to craftwork

D: that refers to craftwork making the meaning of the sentence as a whole awkward: unlike
craftwork A, craftwork B received prestige for their craftwork?

E: same meaning issue as D

In an effort improve the quality of patient care, Dr. Lydia Temoscho is directing one of several
clinical research projects that seek to determine the extent that psychological counseling helps to
supplement the medical treatment of serious disease.

(A) seek to determine the extent that psychological counseling helps to supplement

(B) seek to determine how helpful psychological counseling is in supplementing

(C) seeks to determine how helpful psychological counseling is to supplement

(D) seeks to determine to what extent psychological counseling is a help in supplementing

(E) seeks to determine the extent that psychological counseling is helpful in supplementing
(Egmat): khi c before hoc after th dung th qk hay qk hon thnh l optional

Eg: it happened after the ice age destroyed/had destroyed the worlds life.

Past perfect tense only used when 2 actions are related

Eg: they learned about changes that happened in the ice age wrong because 2 action (learning
vs. happening are not related)

EG: During the war John Chadwick spent time as a cryptanalyst in Alexandria, where he had broken Italian
ciphers before moving to Bletchley Park, where he attacked Japanese ciphers.

A. spent time as a cryptanalyst in Alexandria, where he had broken Italian ciphers before moving
B. had spent time as a cryptanalyst in Alexandria, when he had broken Italian ciphers before being moved
C. spent time as a cryptanalyst in Alexandria, when he had broken Italian ciphers before moving
D. spent time as a cryptanalyst in Alexandria, where he broke Italian ciphers before moving
E. spent time as a cryptanalyst in Alexandria, where he broke Italian ciphers before having been moved

Since before is present in this sentence, no need to use past perfect to show the order of the
actions.

The meaning of the sentence is that JC spent time as a cryptanalyst in Alexandria, and during the
war, he broke something cannot use past perfect. D

A leading figure in the Scottish enlightenment, Adam Smiths two major books are to democratic
capitalism what Marxs Das Kapital is to socialism.

A. Adam Smiths two major books are to democratic capitalism what

B. Adam Smiths two major books are to democratic capitalism like

C. Adam Smiths two major books are to democratic capitalism just as

D. Adam Smith wrote two major books that are to democratic capitalism similar to

E. Adam Smith wrote two major books that are to democratic capitalism what

A+B+C out because figure cannot be books, should refers to Adam himself

D: similar to cannot followed by a clause. It must be followed by a noun.

E: yes! X to Y what Z to A
Machines replacing human labor, there was wide anticipation that the workweek would continue to
become shorter.

A. Machines replacing human labor, there was wide anticipation that

B. When machines replaced human labor, there was wide anticipation

C. As machines replaced human labor, it was widely anticipated that

D. Insofar as machines replaced human labor, it was widely anticipated

E. Human labor being replaced by machines, there was wide anticipation that[/u]

Socrates could have fled from Athens after he was sentenced to death, but he refused to do it.

(A) Socrates could have fled from Athens after he was sentenced to death, but he refused to do it.
(B) After he was sentenced to death, Socrates might have fled from Athens, but he refused to do it.
(C)After he was sentenced to death, Socrates could have fled from Athens, but he refused to do so.
(D) Refusing to flee from Athens, Socrates could have done so after he was sentenced to death.
(E) Socrates could have fled from Athens but refused to after he was sentenced to death.

Meaning: Socrates b tuyn n t hnh, ng c th trn khi Athens nhng li ko lm vy

A + B: it has no reference out

B: might have fled changes the meaning (nh th l ng y bin mt ri, nhng ko r l i u)

D: could have done so: so has no logical antecedent out

E: refused to: to what? Need do so after to. This answer also distorts the meaning
Studies show that young people with higher-than-average blood pressure and their families have a
history of high blood pressure are more lifely than others to develop a severe form of the condition

A. and their families have a history of high blood pressure


B. whose families have a history of high blood pressure
C. and a history of high blood pressure runs in the family
D. whose families have a history of high blood pressure running in them
E. with a history of high blood pressure running in their family

A: fragment problem: read the sentence as a whole:

Studies show that young people with higher-than-average blood pressure and their families have a
history of high blood pressure are more lifely than others to develop a severe form of the condition

= (young people + their families) have a history ARE more no subject for are - out

C:problematic meaning: (young people and a history) are more lifely than others?

C+E: family (sing) all mentioned young people share 1 family? crazy

D: them can refer to people or families ambiguous;


running in them is supposed to modify history of high blood pressure but history cannot run
in a family (a disease can though)

E: with a history modifies pressure illogical

Factory outlet stores, operated by manufacturers, are usually located miles from downtown and
regional shopping centers so as not directly to be competitive against department stores in the same
trading area.

(A) so as not directly to be competitive against


(B) in order for them not to have direct competition with
(C) so that they do not compete directly with
(D) in order that they are not directly competitive against
(E) for the purpose of not competing directly with

A: so as less prefered in GMAT; be competitive should be compete; directly is place as if it is


trying to modify to be, not competitive

B: in order for: wrong idiom

D: in order that: wrong idiom

E: for is wrong idiom here. You do sth WITH a purpose.

"not competing..." isn't a purpose. A purpose is an active intention. You're trying to DO something.
If this idea were phrased in an active wayi.e., with the purpose of avoiding competition with
xxxxthen it would make sense.
170. Because the financial review covered only the last few fiscal years, and therefore the
investigators were unable to determine the extent to possible earlier overpayments.

(A) and therefore the investigators were unable to determine the extent to possible

(B) so therefore the investigators were not capable of determining the possible extent of

(C) therefore the investigators were unable to determine the possible extent of

(D) the investigators were not capable of determining the possible extent of

(E) the investigators were unable to determine the extent of possible

A+B+C: out for because therefore

D: distorts the meaning not capable of = do not have the skill set to do something. The
investigators do not have the skill set to determine because the financial review covered only few
years? nonsensical. The placement of possible also changes the meaning (in comparison with
intended meaning)

138. As sales of cars and light trucks made in North America were declining 13.6 percent in late
February, many analysts conclude that evidence of a recovering automotive market remains slight.

(A) As sales of cars and light trucks made in North America were declining 13.6 percent in late
February, many analysts conclude

(B) Since sales of cars and light trucks made in North America declined 13.6 percent in late
February, and many analysts conclude

(C) With sales of cars and light trucks made in North America declining 13.6 percent in late
February, with many analysts concluding

(D) Because sales of cars and light trucks made in North America declined 13.6 percent in late
February, many analysts concluded

(E) Because of sales of cars and light trucks made in North America declining 13.6 percent in late
February, therefore, many analysts concluded

A: when a decline is showed by a percent (13.6%) its done, its one-time incident, not a progress
anymore. were declining is incorrect.

B: since X declined, AND Y conclude wrong structure.

C: fragment

E: declining modifies North America?

D: CORRECT. it can be like this: they concluded that the market remains slight
The complex tax dispute between the Covered Bridge Mall and Harris Township is not likely to be
adjudicated for several years, and, in the meantime, both sides are intent on creating difficulties for
the other.

A. both sides are intent on creating difficulties for the other

B. both sides are intent on creating difficulties for each other

C. each side is intent on creating difficulties for the other

D. each side are intent on creating difficulties for one another

E. the sides are both intent on creating difficulties for each other

both means 2 entities are going in the same direction, not going against each other A+B+E out

(hence, both is not lined up with the other, other)

D: one another cannot be used for 2 parties only.

As a result of record low temperatures, the water pipes on the third floor froze, which caused the
heads of the sprinkler system to burst, which released torrents of water into offices on the second
floor.

A) which caused the heads of the sprinkler system to burst, which released torrents of water
B) which caused the heads of the sprinkler system to burst and which released torrents of water
C) which caused the heads of the sprinkler system to burst, torrents of water were then released
D) causing heads of sprinkler system to burst, then releasing torrents of water
E) causing heads of the sprinkler system to burst and release torrents of water

A+B+C: using which to modify the action of freezing the pipes is incorrect

C: in this answer, we have 2 clauses joining by just a comma run on sentence

D: meaning: the torrents of water were released not because of the frozen pipes, but because the
sprinkler system bursts. This SUCK!
The computer company reported strong second-quarter earnings that surpassed Wall Streets
estimates and announced the first in a series of price cuts intended to increase sales further.
A. The computer company reported strong second-quarter earnings that surpassed Wall Streets
estimates and announced the first in a series of price cuts intended to increase sales further.
B. The report of the computer company showed strong second-quarter earnings, surpassing Wall
Streets estimates, and they announced the first in a series of price cuts that they intend to increase
sales further.
C. Surpassing Wall Streets estimates, the report of the computer company showed strong second-
quarter earnings, and, for the purpose of increasing sales further, they announced the first in a
series of price cuts.
D. The computer company reported strong second-quarter earnings, while announcing the first in a
series of price cuts for the purpose of increasing sales further that surpass Wall Streets estimates.
E. The computer company, surpassing Wall Streets estimates, reported strong second-quarter
earnings, while announcing that to increase sales further, there would be the first in a series of price
cuts

B+C: they refers to company wrong

C: the report surpasses the estimates? No, the earnings does

D: that refers to what?

E: the company surpasses the estimates? No.

61. Among lower-paid workers, union members are less likely than nonunion members to be
enrolled in lower-end insurance plans imposing stricter limits on medical services and requiring
doctors to see more patients, and spend less time with each.
(A) imposing stricter limits on medical services and requiring doctors to see more patients, and
spend
(B) imposing stricter limits on medical services, requiring doctors to see more patients, and
spending
(C) that impose stricter limits on medical services, require doctors to see more patients, and spend
(D) that impose stricter limits on medical services and require doctors to see more patients,
spending
(E) that impose stricter limits on medical services, requiring doctors to see more patients and
spending

A: , and tries to make a list of imposing, requiring, spend = not //; about meaning, spend less
time with each should be the result of the action requiring not infinitive

B: imposing // requiring // spending implies that the plans spend less time.. nonsensical

C: impose // require // spend implies that the plans spend less time nonsensical

E: requiring // spending implies that the plans spend less time nonsensical (the plans impose
limits, spending less time)

D: ok!
Question in D: how can we say that spending does not modifies plans, but doctors?

Ron: in choice d, you could legitimately make a case that 'spending' could modify the entire huge
clause about what insurance plans do, and is therefore ambiguous. however, that's the OA, so
you've learned that this problem is ok in the eyes of the gmat people. if there's a rule that can be
articulated here, it's probably something along the lines of 'participial modifier applies to nearest
action'.

Question: in D, shouldnt it be: that impose, and that require? Can we omit that? (in E-gmat,
clauses must start with the same words to be //)

Ron: there are two kinds of parallel signals: ONE-PART (such as "and", "or", "but"), and TWO-PART
(such as "not only ... but also", "both ... and").

when you have PARALLELISM WITH A ONE-PART SIGNAL, the only words that are "locked in" are
the ones directly FOLLOWING the signal.

as long as you can find the corresponding structure in the other part, then the parallelism is fine.

examples: i worked in nevada and florida/ i worked in nevada and in florida.

BOTH ARE FINE.

reasons:

in the first, the part that's "locked in" by the signal and is just florida. therefore, the parallel
construction would be just nevada.

since that construction is there, the sentence is parallel:

i worked in (nevada and florida).

in the second, the part that's "locked in" by the signal and is in florida. therefore, the parallel
construction would be just in nevada.

since that construction is there, the sentence is parallel:

i worked (in nevada and in florida).

--

for completely analogous reasons, this sentence would be fine either with or without your second
"that":

an increase that would amount to roughly five miles per gallon and would represent...

an increase

that would amount to roughly five miles per gallon and that would represent...
In preparation for the prediction of a major earthquake that will hit the state, a satellite-based
computer network is being built by the California Office of Emergency Services for
identifying earthquake damage and to pinpoint the most affected areas within two hours of the
event.

(A) In preparation for the prediction of a major earthquake that will hit the state, a satellite-based
computer network is being built by the California Office of Emergency Services for identifying

(B) In preparing for the prediction that a major earthquake will hit the state, the California Office of
Emergency Services is building a satellite-based computer network that will identify

(C) In preparing for a major earthquake that is predicted to hit the state, the California Office of
Emergency Services is building a satellite-based computer network to identify

(D) To prepare for the prediction of a major earthquake hitting the state, a satellite-based computer
network is being built by the California Office of Emergency Services to identify

(E) To prepare for a major earthquake that is predicted to hit the state, the California Office of
Emergency Services is building a satellite-based computer network that will identify

A+C+D: meaning? Prepare for a prediction?

E: will identify and to pinpoint.. not // B is correct

Galileo did not invent the telescope, but on hearing, in 1609, that such an optical instrument had
been made, he quickly built his own device from an organ pipe and spectacle lenses.
A. Galileo did not invent the telescope, but on hearing, in 1609, that such an optical instrument had
been made, he
B. Galileo had not invented the telescope, but when he heard, in 1609, of such an optical instrument
having been made,
C. Galileo, even though he had not invented the telescope, on hearing, in 1609, that such an optical
instrument had been made, he
D. Even though Galileo did not invent the telescope, on hearing, in 1609, that such an optical
instrument had been made,
E. Even though Galileo did not invent the telescope, but when he heard, in 1609, of such an optical
instrument being made, he

B: the subject of the second clause gone missing; had been made should be in place of having..

C: remove all the fluff Galileo, he built the device 2 subjects? For what?

D: again, no subject for the later clause? Crazy.

E: Eventhough but - redundant.

Also: in B having been made is wrong (same as the problem of honeybees stinger..).
The construction: PREP + Noun + Ving is wrong:

Ive never heard of bees stinging dogs wrong

Turning away from literary realism to write romantic stories about the peasant life and landscape of
northern Sweden, in 1909 Selma Lagerl was the novelist who became the first woman and was also
the first Swedish writer to win the Nobel Prize for Literature.

A. Turning away from literary realism to write romantic stories about the peasant life and landscape
of northern Sweden, in 1909 Selma Lagerl was the novelist who became the first woman and was
also the first Swedish writer to win
B. She turned away from literary realism and wrote romantic stories about the peasant life and
landscape of northern Sweden, and novelist Selma Lagerl in 1909 became the first woman as well
as the first Swedish writer that won
C. Selma Lagerl was a novelist who turned away from literary realism to write romantic stories about
the peasant life and landscape of northern Sweden, and in 1909 she became the first woman in
addition to the first Swedish writer winning
D. A novelist who turned away from literary realism to write romantic stories about the peasant life
and landscape of northern Sweden, Selma Lagerl became in 1909 the first woman and also the first
Swedish writer to win
E. As a novelist, Selma Lagerl turned away from literary realism and wrote romantic stories about the
peasant life and landscape of northern Sweden, in 1909 becoming the first woman and also the first
Swedish writer that won

A: 1909 turned away? Silly

B: so she is not Selma? She turned away, and Selma won. odd.

C: in addition to is wrong: she became the first woman (on Eden garden?) in addition to blah

winning should be to win: the first one to do smth

E: turning away from literary realism is the job of a novelist?; that is wrong to refer to people

Responding to the publics fascination with-and sometimes undue alarm over-possible threats from
asteroids, a scale developed by astronomers rates the likelihood that a particular asteroid or comet
may collide with Earth.
A. a scale developed by astronomers rates the likelihood that a particular asteroid or comet may
B. a scale that astronomers have developed rates how likely it is for a particular asteroid or comet to
C. astronomers have developed a scale to rate how likely a particular asteroid or comet will be to
D. astronomers have developed a scale for rating the likelihood that a particular asteroid or comet
will
E. astronomers have developed a scale that rates the likelihood of a particular asteroid or comet that
may

A+B: a scale cannot respond to the publics fascination.


E: likelihood may.. redundant

C: You can't say "likely...will be to". It's gibberish. Your ear would recognize that if there weren't all
those words in between, confusing things. Try a short example. You should always be able to
replace "likely" with "probably".
My program determines how likely you will be to love me.
My program determines how probably you will be to love me...MAKES NO SENSE!

Whereas lines of competition are clearly defined in the more established industries, in the Internet
industry they are blurred and indistinct, as companies that compete one day may be partners the
next. (Gmat Prep verbal section)

A. Whereas lines of competition are clearly defined in the more established industries, in the Internet
industry they are blurred and indistinct, as companies that compete
B. Although the lines of competition are clearly defined in industries that are more established, they
are blurred and indistinct in the Internet industry, as competing companies
C. The lines of competition are clearly defined in the more established industries, unlike the Internet
where they are blurred and indistinct, as companies that compete
D. Unlike more established industries, where the lines of competition are clearly defined, they are
burred and indistinct in the Internet industry, as companies that compete
E. Unlike more established industries, with clearly defined lines of competition, those of the Internet
industry are blurred and indistinct, as competing companies

C: the lines of competitions are, unlike the internet - not compatible

D: unlike the industries., they are they refers to lines of competition which lies in a non-
restrictive clause NO!

E: same as D

Now what takes time is how to eliminate B. since A and B differ in the placement of compete and
companies, take a look at adjacent entities (thats a MUST in such cases as this one): one day
cannot modify companies, it has to modify compete A.

Note: ONE DAY is an adv it can only modify a verb or a clause B is wrong.

Written in ink or engraved by stylus, more than 2,000 letters and documents on wooden tablets
excavated at the site of the old roman fort at vindolanda in northern England are yielding a historical
account of the military garrison in the first and second centuries that are so vivid in their details
about personal life as they are from Pompeii

A. are so vivid in their details about personal life as they are

B. are as vivid in their details of personal life as those gathered

C. is as vivid in their detail about personal life as that gathered

D. is as vivid in its details of personal life as that gathered


E. is so vivid in its details of personal life as is that
More than 300 rivers drain into Siberia's Lake Baikal, which holds 20 percent of the world's fresh
water, more than all the North American Great Lakes combined. (Comparison problem)

(A) More than 300 rivers drain into Siberia's Lake Baikal, which holds 20 percent of the world's fresh
water, more than all the North American Great Lakes combined.

(B) With 20 percent of the world's fresh water, that is more than all the North American Great Lakes
combined, Siberia's Lake Baikal has more than 300 rivers that drain into it.

(C) Siberia's Lake Baikal, with more than 300 rivers draining into it, it holds more of the world's fresh
water than all that of the North American Great Lakes combined, 20 percent.

(D) While more than 300 rivers drain into it, Siberia's lake Baikal holds 20 percent of the world's
fresh water, which is more than all the North American Great Lakes combined.

(E) More than all the North American Great Lakes combined, Siberia's Lake Baikal, with more than
300 rivers draining into it, holds 20 percent of the world's fresh water.

B+C+D: out without saying


D: while but no contrast, no simultenous events? Also in this answer: which refers to water
how can an amount of water be MORE than lakes? by adding which, the modifying objects are
rigidly confined to nouns

E: when we have a modifier that is not a clause (no subject) it modifies the following noun.

In this answer: more than combined modifies Baikal - how can lake A more than lake B? take this
analogy, Toan:

More than John, Jim holds 20 balls nonsense. Because more than John now acts as an adj
(modifier) modifying Jim. How can Jim more than John? He might be taller, bigger, or richer but
definitely not more. Same token, Great lakes cannot be MORE THAN Baikal

It should be: Jim holds 20 balls, more than John (does/holds elided without ambiguity).
Almost like clones in their similarity to one another, the cheetah species homogeneity makes them
especially vulnerable to disease. (comparison problem)

(A) the cheetah species homogeneity makes them especially vulnerable to disease

(B) the cheetah species is especially vulnerable to disease because of its homogeneity

(C) the homogeneity of the cheetah species makes it especially vulnerable to disease

(D) homogeneity makes members of the cheetah species especially vulnerable to disease

(E) members of the cheetah species are especially vulnerable to disease because of their
homogeneity

A+C+D: wrong comparison

Between B and E: the modifier goes: like clones in their similarity to one another must modify a
plural noun E. (seriously, Toan, how can a species similar to one another?)

Providing initial evidence that airports are a larger source of pollution than they were once believed
to be, environmentalists in Chicago report that the total amount of pollutant emitted annually by
vehicles at the O'hare International airport is twice as much as that which is being emitted annually
by allmotor vehicles in the Chicago Metro area

A) as much as that which is being emitted annually by all


B) as much annually as is emitted by the
C) as much compared to what is annually emitted by all
D) that emitted annually by all
E) that emitted annually compared to

Unlike mainstream American businesses, more than half of which fail, the failure rate for businesses
in the Amish community hovers around 5 percent.
A. Unlike mainstream American businesses, more than half of which fail
B. Unlike mainstream American businesses, in which the failure rate is more than half
C. Unlike mainstream American businesses, where more than half of them fail
D. While the rate of mainstream American businesses failing is more than half
E. While more than half of mainstream American businesses fail
A+B+C: comparing business vs. rate

D: you cannot say: the rate is half. Rate = 50% - ok

Also in D: Prep + N + Ving: dead wrong

More rate?
As sources of electrical power, windmills now account for only about 2,500 megawatts nationwide,
but production is almost expected to double by the end of the year, which would provide enough
electricity for 1.3 million households.

A. almost expected to double by the end of the year, which would provide

B. almost expected that it will double by the end of the year, thus providing

C. expected that it will almost double by the end of the year to provide

D. expected almost to double by the end of the year and thus to provide

E. expected almost to double by the end of the year, which would thus be providing

A+B: almost expected? Really?

A+E: which modifies year?

B+C: it can only refer back to production production is expected that production will double?

C: that introduces new clause the previous clause is incompleted: ..production is expected

Not one of the potential investors is expected to make an offer to buy First Interstate Bank until a
merger agreement is signed that includes a provision for penalties if the deal were not to be
concluded. (*)

A. is expected to make an offer to buy First Interstate Bank until a merger agreement is signed that
includes a provision for penalties if the deal were

B. is expected to make an offer for buying First Interstate Bank until they sign a merger agreement
including a provision for penalties if the deal was

C. is expected to make an offer to buy First Interstate Bank until a merger agreement be signed by
them with a provision for penalties if the deal were

D. are expected to make an offer for buying First Interstate Bank until it signs a merger agreement
with a provision for penalties included if the deal was

E. are expected to be making an offer to buy First Interstate Bank until they sign a merger
agreement including a provision for penalties if the deal were

Very tricky one.

D+E: are

B: if was

C: by them is ambigous and wordy; sounds like the provision is not inside of the agreement
A: that here is ok. That refers to agreement, but if we placed it near agreement, is signed would
have to move towards the end of the sentence hard to follow. (exception of the touchhas rule)
A long-term study of some 1,000 physicians indicates that the more coffee these doctors drank, the
more they had a likelihood of coronary disease.

(A) more they had a likelihood of coronary disease

(B) more was their likelihood of having coronary disease

(C) more they would have a likelihood to have coronary disease

(D) greater was their likelihood of having coronary disease

(E) greater was coronary disease likely

A: they had the doctors possess the likelihood? Illogical.

More should not be used to describe likelihood. greater should do the trick.

'likelihood' means 'the probability of a specified outcome'. Thus, it's imprecise to say they had a
likelihood/probability of coronary disease. The correct usage should be "likelihood of having
coronary disease'.

B+C: more is wrong.

E: greater likely is wrong should be more likely

* if you are dealing with a word that literally describes a numerical quantity (number, rate, height,
length, volume, population, etc.), then you should use greater.

The population of Colombia is greater than that of Argentina --> correct

The population of Colombia is more than that of Argentina --> incorrect

* if you are dealing with a word that describes some abstract characteristic of something (power,
influence, effect, etc.), then you can use a/an + greater or "more" (the latter WITHOUT 'a/an').

John's father has had a greater influence on the development of his personality than has his mother.
--> correct

The Prime Minister has more power than the King, even though the latter is the country's titular
leader. --> correct

* if you are dealing with a word that describes things that you can actually count, then you should
use more.

I have more sports jerseys than anyone else I know --> correct
* if you are dealing with an uncountable noun that is NOT a numerical quantity, then you should use
more.

there is more furniture in this store than in the other one --> correct
Scientists have dated sharp-edged flakes of stone found in the fine-grained sediments of a dry
riverbed in the Afar region of Ethiopia to between 2.52 and 2.60 million years ago, pushing back by
more than 150,000 years the earliest date when it is known that humans made stone tools.

(A) when it is known that humans made

(B) at which it is known that humans had made

(C) at which humans are known to have made

(D) that humans are known to be making

(E) of humans who were known to make

D: date that - wrong modifier

E: distort the logical meaning

A+B: imply that the fact (that humans made stone tools) was known at that date (distort the
meaning) in the correct answer, the "at which" is followed by a clause whose subject is "humans",
and which describes the actual action that took place at that date. therefore, i think the idea is that
this clause more accurately describes the chronology of the events: i.e., the toolmaking happened at
that date, and the fact is known now.

since 'to V' is not a verb, it doesn't have a tense of its own. rather, it is relative to the timeframe of the
main sentence.
'to V' = SAME timeframe as main sentence
'to have V-ed' = EARLIER than the main sentence

(A) when it is known that humans made

(B) at which it is known that humans had made

these sentences are talking about when something became known. we SHOULD be talking
about when people made tools! (as in C)

According to analysts, an alliance between three major personal computer companies and most of
the nations largest local telephone companies would enable customers to receive Internet data over
regular telephone lines with speeds much higher than is currently possible.
A. with speeds much higher than is
B. with speeds that are much higher than are
C. at much higher speeds as are
D. at much higher speeds than that
E. at speeds much higher than are

A+B: you get data AT speeds, not with speeds

C: higher as?
D: that refers to speeds wrong
Heirloom tomatoes, grown from seeds saved from the previous year, only look less appetizing than
their round and red supermarket cousins, often green and striped, or have plenty of bumps and
bruises, but are more flavorful.

(A) cousins, often green and striped, or have plenty of bumps and bruises, but are

(B) cousins, often green and striped, or with plenty of bumps and bruises, although

(C) cousins, often green and striped, or they have plenty of bumps and bruises, although they are

(D) cousins; they are often green and striped, or with plenty of bumps and bruises, although

(E) cousins; they are often green and striped, or have plenty of bumps and bruises, but they are

A+B+C: wrong placement of modifier. It should be near heirloom potatoes

D: or with not // with previous clause; although should always be followed by a clause.

After several years of rapid growth, the health care company became one of the largest health care
providers in the metropolitan area, while it then proved unable to handle the increase in business,
falling months behind in its payment to doctors and hospitals.

A. while it then proved unable to handle the increase in business, falling months behind in its
payment to

B. while it then proved unable to handle the increase in business and fell months behind in its
payment to

C. but then it proved unable to handle the increase in business, falling months behind in its paying

D. but then proving unable to handle the increase in business, falling months behind in paying

E. but then proved unable to handle the increase in business, falling months behind in paying

A+B: while can use to show the contrast, but should use to contrast between 2 simultinous things.

C: in its paying is wrong in this context, gerund is not accurate since we have payment. Gerund
here may act as an adj modifying doctors C reads: the compay falls behind its doctors.

E: proving not //
Under high pressure and intense heat, graphite, the most stable form of pure carbon, changes into
the substance commonly referred to as diamond and remaining this way whether or not the heat
and pressure are removed.

(A) remaining this way whether or not

(B) remaining like that even as

(C) remaining as such whether or not

(D) remains in this way although

(E) remains thus even when

A+B+C: not //

D: in this way is wrong since. because no "way" has been described. in fact, no adverb is present
at all to be properly parallel to "in this way" (an adverbial construction); diamond is most certainly
not a "way" of doing anything.

D: Although shows the contrast between 2 facts. This sentence shows hypothetic condition
(pressure are removed) not correct.

Unearthed in China, fossils of feathered dinosaurs offer the most dramatic evidence yet discovered
of the close evolutionary relationship between dinosaurs and birds.

A. offer the most dramatic evidence yet discovered of the close evolutionary relationship between
dinosaurs and birds

B. offer evidence more dramatic than what has yet been discovered of the close evolutionary
relationship between dinosaurs and birds

C. offer more dramatic evidence of the close evolutionary relationship than any yet discovered
between dinosaurs and birds

D. have offered the most dramatic evidence of the close evolutionary relationship between dinosaurs
and birds that have yet been discovered

E. have offered more dramatic evidence than any that has yet been discovered of the close
evolutionary relationship between dinosaurs and birds

B+C+E: -What has yet been discovered- should naturally exclude the discovery under issue, so
that a comparison could be made. Here it is not. You must add the word other- and state - than
what other has yet been discovered (this discovery is one of the bunch cannot say: it is
more dramatic than itself. MOST should be better)
D: have yet been discovered wrongfully modify dinosaurs and birds)
Nobody knows exactly how many languages there are in the world, partly because of the difficulty of
distinguishing between a language and the sub-languages or dialects within it, but those who have
tried to count typically have found about five thousand.

A. and the sub-languages or dialects within it, but those who have tried to count typically have
found

B. and the sub-languages or dialects within them, with those who have tried counting typically
finding

C. and the sub-languages or dialects within it, but those who have tried counting it typically find

D. or the sub-languages or dialects within them, but those who tried to count them typically found

E. or the sub-languages or dialects within them, with those who have tried to count typically finding

D+E: between or - out!

B: between a language and dialects within them??? What are them?

C: second it is crazy since it is singular, do you still have to count, Toan? Not worth considering.

Discussion of greenhouse effects has usually focused on whether the Earth would warm and by how
much, but climatologists have indicated all along that the most obvious effects, and those that
would have the largest impact on people, would be extremes of temperature, precipitation, and
storminess.

A. the most obvious effects, and those that would have the largest impact on people, would be
extremes of temperature, precipitation, and storminess

B. the effects that are the most obvious ones, extremes of temperature, precipitation, and
storminess, would be those impacting the most on people

C. those effects to have the largest impact on people, extremes of temperature, precipitation, and
storminess, are what are the most obvious effects

D. extremes of temperature, precipitation, and storminess, the most obvious effects, that they
would have the largest impact on people

E. extremes of temperature, precipitation, and storminess, which are the most obvious effects, are
those to impact the most on people

B: wordy; impacting on people = wrong. Have an impact on s.o/ impact s.o are both ok
B changes the meaning significantly

C: extremes of.. should be appositive modifier modifying effects, but in this sentence, it seems like
extremes is modifying people. are what are = crazy

D: they refers to what? There are so many plural nouns before it. Removing the fluff, the sentence
reads: climatologists indicate that extremes that they would have redundant

E: which cannot refer to a list of nouns. Impact on people is wrong.

A Harvard anthropologist has proposed that using fire to cook food could be dated back to almost
two million years and that it could explain hominid features like having a large brain and small teeth.

A. using fire to cook food could be dated back to almost two million years and that it could explain
hominid features like having

B. the use of fire to cook food could date back almost two million years and could explain such
hominid features as

C. cooking food with fire could date back to almost two million Years, explaining hominid features
like

D. fire used to cook food could date back almost two million years ,explaining hominid features such
as having.

E. fire used for cooking food could be dated back to almost two million years and explain hominid
features like.

D+E: Fire could be dated back? Can we track down the fire? Even if we can track It down, fire
cannot explain hominid features. The use of fire can, though.

A+C+E: like cannot introduce examples.

The army cutworm moth is a critical source of fat for many of Yellowstone National Parks grizzly
bears; they overturn rocks to find them and consuming as many as 40,000 apiece in a single day.

A. bears; they overturn rocks to find them and consuming as many as

B. bears; overturning rocks to find the insects, consuming up to

C. bears, overturning rocks to find them and they consume as many as

D. bears, and they overturn rocks to find them and consume up to


E. bears, which overturn rocks to find the insects, consuming as many as
Traffic safety officials predict that drivers will be equally likely to exceed the proposed speed limit as
the current one.

A. equally likely to exceed the proposed speed limit as

B. equally likely to exceed the proposed speed limit as they are

C. equally likely that they will exceed the proposed speed limit as

D. as likely that they will exceed the proposed speed limit as

E. as likely to exceed the proposed speed limit as they are.

A+B+C: equally as - incorrect

D: 2 will showed up in the sentence duplicate: drivers will be as likely that they will exceed the
new as they are the current

Providing initial evidence that airports are a larger source of pollution than they were once believed
to be,environmentalists in Chicago report that the total amount of pollutant emitted annually by
vehicles at O'hare International Airport is twice as that which is being emitted annually by all motor
vehicles in the Chicago metropolitan area.

A.as that which is being emitted annually by all

B.as much annually as is emitted by the

C.as much compared to what is annually emitted y all

D.that emitted annually by all

E.that emitted annually compared to the

A: that which redundant; being emitted is wordy

B: annually is wrongfully placed in this order redundant issue.

C: a comparison starts with as should end with as, not compared to

E: wrong comparison: emission with vehicles, wrong placement of annually


Scientists claim that the discovery of the first authenticated mammal bones in amber could provide
important clues of determining, in addition to how, when mammals colonized the islands of the West
Indies.

A: of determining, in addition to how, when mammals colonized the islands of the West Indies

B:in the determination of how and when the islands of the West Indies were colonized by mammals

C:to determine how mammals colonized the islands of the West Indies and when they did

D:for determining when the islands of the West Indies were colonized by mammals and how they
were

E:for determining how and when mammals colonized the islands of the West Indies

C+D: very ambiguous they + did/were did what? Were what?

A: clues of determining wrong; in addition to how wrong

Between B&E: E is better with active voice, more concise.

The remarkable similarity of Thule artifacts throughout a vast region can,in part,be explained as a
very rapid movement of people from one end of North America to the other.

A. The remarkable similarity of Thule artifacts throughout a vast region can, in part, be explained as

B. Thule artifacts being remarkably similar throughout a vast region, one explanation is

C. That Thule artifacts are remarkably similar throughout a vast region is, in part, explainable as

D. One explanation for the remarkable similarity of Thule artifacts throughout a vast region is that
there was

E. Throughout a vast region Thule artifacts are remarkably similar, with one explanation for this
being

Most of Portugals 250,000 university students boycotted classes in a one-day strike to protest a law
that requires them to contribute $330 a year toward the cost of higher education, previously paying
$7 per year.
A. year toward the cost of higher education, previously paying $7 per year
B. year toward the cost of higher education, for which was previously paid $7 per year
C. year, compared to the previously $7 per year, toward the cost of higher education
D. year toward the cost of higher education, instead of the $7 per year required previously
E. year as opposed to the $7 per year required previously for the cost of higher education

A: Previously paying modifies students, not the entire clause it was placed incorrectly

B: for which was previously paid makes no sense! Who paid?


C: the Previously $7 per year? previously modifies a noun? - out

E: not clear for what $330 a year is contributed


Section 301 of the 1988 Omnibus Trade and Competitiveness Act enables the United States Trade
Representative to single out a country as an unfair trader, begin trade negotiations with that country,
and, if the negotiations do not conclude by the United States government's being satisfied, to
imposesanctions.
A. by the United States government's being satisfied, to impose
B. by the United States government's satisfaction, impose
C. with the United States government's being satisfied, imposing
D. to the United States government's satisfaction, impose
E. to the United States government's satisfaction, imposing

Idiom: do something to someones satisfaction D is correct

Scientists have recently discovered that the ultrathin, layered construction of a butterflys wings, the
same as the one making some butterflies shimmer via the phenomenon of iridescence, are enabling
the insect to control how much heat energy is absorbed by its wings and how much is reflected
away.

A. wings, the same as the one making some butterflies shimmer via the phenomenonof iridescence,
are enabling

B. wings, which is the same one that makes some butterflies shimmer via thephenomenon of
ridescence, that also enables

C. wings is the same as the one that makes some butterflies shimmer via the phenomenon of
ridescence, enabling

D. wingsthe same construction that makes some butterflies shimmer via the phenomenon of
iridescencealso enables

E. wingsof the same construction that makes some butterflies shimmer via the phenomenon of
iridescencealso enable

A+E: SV do not agree

B: fragment

C: no comma before is meaning: someone discovers that Ultrathin, the construction (..) is ()
not valid construction. Also, , enabling cannot logically modify preceding clause.

The three women, liberal activists who strongly support legislation in favor of civil rights and
environmental protection, have consistently received labor's unqualifying support.

a. have consistently received labor's unqualifying support


b. are consistetly receiving the unqualifying support of labor
c. have consistently received the unqualified support of labor
d. receive consistent and unqualified support by labor
e. are receiving consistent and unqualified support by labor
A+B: unqualifying DOES NOT EXIST its not an adjective should be gerund the support is
unqualifying sth? Nonsense. out

D+E: support by labor is wrong.

By merging its two publishing division, the company will increase their share of the country's $21
billion book market from 6 percent to 10 percent, a market ranging from obscure textbooks to mass-
market paperbacks.

A. their share of the country's $21 billion b ook market from 6 percent to 10 percent, a market
ranging

B. from 6 percent to 10 percent its share of the $21 billion book market in the country, which ranges

C. to 10 percent from 6 percent in their share of the $21 billion book market in the country, a
market ranging

D. in its share, from 6 percent to 10 percent, of the $21 billion book market in the country, which
ranges

E. to 10 percent from 6 percent its share of the country's $21 billion book market, which ranges

B: which in this sentence refers to country the closest eligible noun incorrect

A recent review of pay scales indicates that CEOs now earn an average of 419 times more pay than
blue-collar workers, compared to a ratio of 42 times in 1980.

A. that CEOs now earn an average of 419 times more pay than blue-collar workers, compared to a
ratio of 42 times
B. that, on average, CEOs now earn 419 times the pay of blue-collar workers, a ratio that compares
to 42 times
C. that, on average, CEOs now earn 419 times the pay of blue-collar workers, as compared to 42
times their pay, the ratio
D. CEOs who now earn on average 419 times more pay than blue-collar workers, as compared to
42 times their pay, the ratio
E. CEOs now earning an average of 419 times the pay of blue-collar workers, compared to the ratio
of 42 times

Take away: CEOs = CEOs. Dont question!

D+E: the review cannot indicate CEOs. It can only indicate something (fact)

A: a ratio of 42 times is redundant, should be 42 times. This expression is also ambiguous: 42


times what?
B: a ratio that compares to = wrong. As compared to/with is the idiom

42 times what? You cannot hang times like this.


To meet the rapid rising market demand for fish and seafood, suppliers are growing fish twice as
fast as their natural growth rate, cutting their feed allotment by nearly half and raising them on
special diet.

(A) their natural growth rate, cutting their feed allotment

(B) their natural growth rate, their feed allotment cut

(C) growing them naturally, cutting their feed allotment

(D) they grow naturally, cutting their feed allotment

(E) they grow naturally, with their feed allotment cut

A+B: There are two pieces here, and they should be interchangeable (grammatically). Right now,
this says "suppliers are growing fish twice as fast as their natural growth rate". That's a verb "are
growing" compared to a noun "their natural growth rate". You can't do that.

By the way, A rate cannot be FAST.

C: you cant grow fish naturally (nobody needs help growing naturally)

E: not //

The Achaemenid empire of Persia reached the Indus valley in the fifth century B.C, bringing the
Aramaic script with it, from which was derived both northern and southern Indian alphabets.

A) the Aramaic script with it, from which was derived both northern and

B) the Aramaic script with it, and from which deriving both the northern and the

C) with it the Aramaic script, from which derive both the northern and the

D) with it the Aramaic script, from which derives both northern and

E) with it the Aramaic script, and deriving from it both the northern and the

Compared with southern New England's more developed beaches, Maine seems like they have not
dramatically changed in the past few centuries.

A Maine seems like they have not

B Maine's seem not to have


C Maine beaches seems like it has not

D the beaches of Maine have not seem, not exactly have not changed

E Maine seems as if it has


Many of todays mathematicians use computers to test cases that are either too time-consuming or
involve too many variables to test manually, allowing the exploration of theoretical issues that were
impossible to test a generation ago.

A/ are either too time-consuming or involve too many variables to test manually, allowing the
exploration of

B/ either take too much time or involve too many variables to be tested manually; allowing the
mathematicians to explore

C/ would either take too much time or involve too many variables to test manually, allowing them to
explore

D/ would either be too time-consuming or would involve too many variables to test manually; this
capability allows the mathematicians to explore

E/ take too much time or variables to test manually; this capability allows the mathematicians to
explore

Note: E MUCH can only modify time, while variables is countable, many should do the trick.

Employment costs rose 2.8 percent in the 12 months that ended in September, slightly less than
they did in the year that ended in the previous quarter.

(A) less than they did


(B) less than it did
(C) less than they were
(D) lower than
(E) lower than they were

The relevant amount is not the "employment costs" but the percent by which the costs rose. I think
"less than" more clearly conveys the idea that employment costs rose 2.8% in the year ending in
September, while employment costs rose <2.8% in the year ending in June.
The hognose snake puts on an impressive bluff, hissing and rearing back, broadens the flesh behind
its head the way a cobra does, feigning repeated strikes, but having no dangerous fangs and no
venom, eventually, if its pursuer is not cowered by the performance, will fall over and play dead.

A broadens the flesh behind its head the way a cobra does, feigning repeated strikes, but having no
dangerous fangs and no venom,

B broadens the flesh behind its head the way a cobra does and feigns repeated strikes, but with no
dangerous fangs and no venom,

C broadening the flesh behind its head the way a cobra does and feigning repeated strikes, but it
has no dangerous fangs and no venom, and

D broadening the flesh behind its head the way a cobra does and feigns repeated strikes, but with
no dangerous fangs and no venom, and

E broadening the flesh behind its head the way a cobra does, feigning repeated strikes, but with no
dangerous fangs and no venom, and

Based on meaning:

(1) the snake puts on a bluff hissing + rearing Back + broadening the flesh + feigning strikes.
(2) It has no fang will fall and play dead.
A+B: (aside from meaning issue) the author tries to list 3 things: (puts..., broadens..., will fall...) in
parallel. But this effort is wrong:

there are exactly two ways to put 3 things into a list in the english language:
X, Y, and Z
X, Y, or Z
...and that's it.
there's no such construction as "X, Y, but Z", so, both of the first two choices are gone.

A+B+D+E: having no dangerous fangs and no venom has no reference.

One report concludes that many schools do not have, or likely to have, enough computers to use
them effectively.
a. same
b. nor
c. or are
d. nor are they
e. nor are not

Remember: BE LIKELY + TO V
NOTE:
Noun 1 + (not) V1 + NOR + helping verb + Noun 2 is ok: schools do not have nor are they likely to
have.. OK
N1 + NOT V + NOR + clause WRONG!!!!: schools do not have nor they are likely to have
A recent and popular self-help book wryly notes that if adolescence was not so painful, it would have a
droll comedic aspect, at least in retrospect.

A/ was not so painful, it

B/ was not so painful, they

C/ were not so painful, they

D/ were not so painful, it

E/ were not so painful, being one

Subjunctive mood adolescence WERE not so painful, IT would have D

The new house's front door is secured by an electronic lock; a feature that not only allows residents to
lock and unlock the door without physical keys, but also may prevent them from entering the house
during a power outage.

A/ a feature that not only allows residents to lock and unlock the door without physical keys, but also
may prevent

B/ a feature allowing residents without a physical key to lock or unlock the door, but also possibly
preventing

C/ a feature allows residents locking or unlocking the door without physical keys, but also possibly
preventing

D/ this feature allows residents to lock and unlock the door without a physical key, but may also prevent

E/ this feature not only allows residents without physical keys to lock or unlock the door, but also
prevents

A semicolon introduces a new clause A+B out (fragment)

C: alows locking wrong

E: without physical keys wrongly placed modifies residents instead of the action.

Not only but also must be used to indicate that two ideas reinforce each other; its usage doesnt
make sense for two contrasting ideas.
D: The use of but (also), without not only, correctly indicates the contrast between the two parallel
elements. Allows and may prevent are parallel.

Vous aimerez peut-être aussi